Сохранен 516
https://2ch.hk/spc/res/127536.html
Домены arhivach.top и arhivach.site временно не функционируют! Используйте домен ARHIVACH.XYZ.
24 декабря Архивач восстановлен после серьёзной аварии. К сожалению, значительная часть сохранённых изображений и видео была потеряна. Подробности случившегося. Мы призываем всех неравнодушных помочь нам с восстановлением утраченного контента!

Тред всяческих вопросов №17.

 Аноним 03/10/14 Птн 01:57:50 #1 №127536 
14122870704110.jpg
Тред разнообразных вопросов о Вселенной, жизни и всего такого.

Спрашиваем то, за что в других местах выдают путёвку в биореактор. Здесь анонимные ученые мирового уровня критически рассмотрят любые гениальные идеи и нарисованные в Paint схемы, расскажут про пузырь Альбукерке, объяснят из чего сделаны черные дыры и что же будет, если включить фонарик на скорости света.

Прошлый тред тут: https://2ch.hk/spc/res/122588.html

The Cosmos is all that is or was or ever will be.
Аноним 03/10/14 Птн 05:23:17 #2 №127539 
>>127536
Свет будет отставать, это если на сверхсветовой, а если на просто световой то лучи будут стоять на месте
Аноним 03/10/14 Птн 05:25:43 #3 №127540 
Есть симуляторы лучше и интереснее чем kerbal space и оrbiter?
Аноним 03/10/14 Птн 05:28:49 #4 №127541 
Что такое горизонт событий простыми словами, чтобы можно было представить в воображении?
Аноним 03/10/14 Птн 08:07:47 #5 №127543 
>>127540
NASA
Аноним 03/10/14 Птн 08:45:44 #6 №127544 
>>127541
Граница зоны невозврата
>>127540
Майнкрафт
Аноним 03/10/14 Птн 09:28:35 #7 №127546 
>>127540
А что для тебя лучше и интереснее?
Если посложнее - ИРЛ.
Если попроще и динамичнее, то пиздуй отсюда.
Аноним 04/10/14 Суб 04:16:05 #8 №127674 
>>127541
>Что такое горизонт событий простыми словами
Условная граница, из-за которой ни что не может вернуться во внешний мир, даже свет, даже небо, даже Аллах.
Аноним 04/10/14 Суб 14:16:56 #9 №127707 
>>127540
Есть space engineers с лучшей на сегодняшний день моделью конструирования трения и повреждений. Физика - ньютонка с верхним лимитом скорости, но есть мод на неполную релятивисткую, где этот верхний лимит становится ассимптотическим пределом. Ну и лучший мультиплеер на сегодня.
Аноним 05/10/14 Вск 00:34:48 #10 №127784 
14124548884040.jpg
Каких размеров рисовать хуй,чтоб увидеть его с мкс?
Аноним 05/10/14 Вск 02:32:54 #11 №127786 
14124619745990.png
>>127541
Поверхность за которую наблюдатель не может заглянуть из-за релятивистских эффектов.
В первом случае это поверхность отделяющая наблюдаемую вселенную от ненаблюдаемой, свет от которой не может дойти до нас из-за расширения пространства, т.к. пространство расширяется в каждой точке, то это те области, которые удаляются от нас со сверхсветовой скоростью.
Во втором случае это поверхность чёрной дыры, которую не может покинуть свет, поверхность для которой время остановилось. Во время коллапса ЧД гравитация увеличивается и время замедляется, коллапс происходит всё медленнее и медленнее, пока размер ЧД не начнёт асимптотически приближаться к сфере Шварцшильда. При этом достигнуть размеров СШ ЧД сможет лишь за бесконечное время, потому ЧД очень медленно, но излучает фотоны с очень большой длиной волны (наногерцы, скажем). Кроме того, электрон-позитронные пары рождённые возле дыры неодинаково ведут себя, так позитроны падают на дыру, а электроны медленно испаряются с поверхности дыры за пределами СШ. В результате масса дыры уменьшается из-за аннигиляции её вещества с антивеществом. В результате в пределе ЧД уменьшится и исчезнет.
Для наблюдателя внутри ЧД это будет выглядеть как коллапс, затем мгновенный прилёт античастиц и излучение частиц и полная аннигиляция ЧД.
Аноним 05/10/14 Вск 04:54:11 #12 №127787 
14124704510820.jpg
>>127784
Пикрилейтеда достаточно, с условием что он будет ярко светиться. Мы же видим освещенную МКС невооруженным взглядом ночью, вот и они его увидят, он приблизительно тех же размеров.
Аноним 05/10/14 Вск 07:39:10 #13 №127788 
>>127784
Что бы четко понять, что это именно хуй, а не что иное - этого >>127787 размера явно не достаточно.
С МКС хорошо видны только довольно крупные, хорошо различимые объекты. Структура Ришат, например.
Аноним 05/10/14 Вск 07:55:53 #14 №127789 
>>127786
> с очень большой длиной волны (наногерцы, скажем)
>длина волны
>герц
Уносите этого макиного астрофизика.
Аноним 05/10/14 Вск 13:38:43 #15 №127802 
>>127789
А что не так? Сверхдлинноволновое излучение будет иметь частоты в наногерцы. Он ебал её рука и вероятно, хотел написать что-то вроде наногерцевые. Сам под риталином изъясняюсь как мудак.
Аноним 05/10/14 Вск 16:17:50 #16 №127807 
>>127802
>А что не так?
А то что длинна волны - это расстояние между "гребнями" измеряется в метрах, а частота -количество повторяющихся событий в секунду, в герцах.
Аноним 05/10/14 Вск 18:35:13 #17 №127822 
Читал один фантастический романьчик вот абзац оттуда.
Вам что-нибудь известно о новой звезде в созвездии Орла?

Дэв откинулся назад и начал просматривать банки данных своей оперативной памяти. Да, ему удалось найти кое-что по этому вопросу, отрывочные сведения, полученные много лет назад, ещё в те времена, когда он читал обо всём, что касалось звёзд. Он знал, что на земном небосклоне существует область в созвездии Орла, где одно время было зафиксировано необычно много новых, т.е. взрывающихся звезд. В течение одного сорокалетнего периода в первой половине XX века двадцать пять процентов всех новых звезд, наблюдаемых с Земли, появились в районе, равном одной части процента всего неба; только в одном 1936 году там появились сразу две новые, а Новая звезда Орла 1918 года стала самой яркой из всех звезд, зарегистрированных на протяжении трёх столетий и затмила все остальные звезды небосвода, кроме Сириуса
Аноны это правда , и если правда чем обясняют кроме случайного совпадения .
Аноним 05/10/14 Вск 18:44:08 #18 №127824 
>>127822
Честно говоря не знаю, и выглядит немного неправдоподобно, уж слишком всё-таки высокая частота...
Но в целом, в общем-то, регионы активного звездообразования - это не совпадение, и даже не норма, а скорее правило. Большинство звёзд формируется в таких регионах. Сами же регионы активного звёздообразования - это результат столкновения двух газовых облаков, либо результат взрывов сверхновых в газовом облаке - в общем, всё, что увеличивает температуру и создаёт градиент плотности.
Аноним 05/10/14 Вск 18:53:19 #19 №127826 
>>127824
>>127822
Самофикс.
А хотя да, там походу речь не о новых, зарождающихся, а о новах - умирающих. Всё-таки новые не особенно ярко вспыхивают. Тогда выглядит выдумкой. Да и самая яркая звезда - Сириус из Большого Пса. Ну и сколько нибудь заметные новы вспыхивают только раз в... не знаю, лет сто-двести.
Аноним 05/10/14 Вск 19:40:25 #20 №127839 
14125236254480.jpg
>>127822
> о новой звезде в созвездии Орла?
https://ru.wikipedia.org/wiki/V603_%D0%9E%D1%80%D0%BB%D0%B0
Аноним 05/10/14 Вск 22:44:39 #21 №127864 
>>127807
Ебать знатоки собрались. На лету герцы в кккилометры переводить не можешь? В обратных сантиметрах спектры не читаешь?
Аноним 05/10/14 Вск 23:27:23 #22 №127875 
>>127864
Ебать ты даун.
Аноним 06/10/14 Пнд 03:07:15 #23 №127881 
>>127807
ЭМ волны вполне можно измерять и в герцах, т.к. световая волна имеет частоту. В зависимости от того, в каком конкретном случае или для каких расчётов, используется длина волны или частота.
Аноним 07/10/14 Втр 00:29:13 #24 №127939 
14126273539090.jpg
Sup спейсаны.
Поясните, возможно ли существование системы планет, как на пик-рилейтед, или бы одну из них распидорасило гравитацией?
Аноним 07/10/14 Втр 01:56:25 #25 №127956 
>>127939
>бы одну из них распидорасило гравитацией
Скорее всего на куски с образованием кольца и обломков, которые выпадая на поверхность большой планеты снесут нахуй эти светящиеся города.

Если даже не распидорасит в куски, то на таком расстоянии приливные силы должны шатать недра и внутренности более малых "планет". Взгляни на Ио для примера что будет.
Аноним 07/10/14 Втр 03:20:51 #26 №127982 
>>127939
Скорее всего нет, судя по линии горизонта, планета, с которой снимают довольно крупных размеров, от чего уже эту систему будет пидорасить, что делает небольшой спутник между ними - отдельный разговор, из этой системы его выпезднет моментально, как из пращи, это не считая того, что он вообще не мог сформироваться на этой орбите.
Короч, картинка - полная хуита, хоть и довольно приятная глазу.
Аноним 07/10/14 Втр 04:56:54 #27 №128013 
>>127939
> горизонт завален
говорить не о чем
Аноним 07/10/14 Втр 11:49:28 #28 №128018 
Звездочёты, поясните даунёнку, умоляю.

Нагуглил карты звездного неба для северного и южного полушария.

Когда корабль пересекает Экватор, какие созвездия можно увидеть с его палубы, северного или южного полушария?
Аноним 07/10/14 Втр 12:06:53 #29 №128019 
>>128018
Если пересекаешь с севера на юг, то при пересечении звёзды постепеноо "перестраиваются"с северного, на южный. Если обратно - то наоборот. Вообще, это одни и те же звёзды, просто выглядят по-разному расположенными с разных полушарий.
Аноним 07/10/14 Втр 12:19:33 #30 №128020 
>>128019
>Вообще, это одни и те же звёзды, просто выглядят по-разному расположенными с разных полушарий.
Ты, судя по всему, вообще без полушарий родился.
>Когда корабль пересекает Экватор, какие созвездия можно увидеть с его палубы, северного или южного полушария?
Видна часть звезд южного и часть северного полушария. Еще зависит от времени года, так как Земеляшка болтается туда-сюда.
Аноним 07/10/14 Втр 12:40:50 #31 №128023 
14126712506270.jpg
>>128018
Смари карочи.
Есть глобус. На нем есть полушарики. Это земляные полушарики.

Есть звездное небо. На нем тоже есть полушария. Но есть один нюанс.
Небесный экватор наклонен к земному на примерно 23.5 градуса цельсия.

Поэтому из северного полушария земляшки можно наблюдать разные т.н. южные созвездия в разные времена года.

> северного или южного полушария?
не "или".

И, десу.
Аноним 07/10/14 Втр 14:25:03 #32 №128027 
>>128018
А чтобы лучше понять, что вообще из себя представляет карта звёздного неба в виде такой вот проекции двух полушарий, как у тебя, представь себе глобус. Но представь, что ты смотришь на него не снаружи, а изнутри. И звёзды на нём нарисованы изнутри - это и есть наше звёздное небо. Потом разрежь его вдоль экватора на две половинки и "расплющь" (а точнее, сделай проекцию на двумерное пространство) каждую из них - получится твоя карта.
Аноним 07/10/14 Втр 16:23:07 #33 №128044 
>>128020
>Видна часть звезд южного и часть северного полушария. Еще зависит от времени года, так как Земеляшка болтается туда-сюда.
В зависимости от времён года туда-сюда болтается Солнце, а не созвездия. Так, на экваторе будут видны вообще все звёзды (кроме совсем полярных), вот только половина из них будет светить днём, а потому их не увидеть.
Аноним 07/10/14 Втр 16:24:16 #34 №128045 
14126846560300.jpg
>>128023
>Небесный экватор наклонен к земному на примерно 23.5 градуса цельсия.
И это пейсач, пиздец, просто пиздец.
Аноним 07/10/14 Втр 17:06:26 #35 №128053 
>>128045
>И это пейсач, пиздец
Блин, конечно же имелось ввиду Кельвина.
Аноним 07/10/14 Втр 17:08:13 #36 №128055 
>>128044
>В зависимости от времён года туда-сюда болтается Солнце
Уносите этого геоцентриста.
Аноним 07/10/14 Втр 17:27:57 #37 №128058 
Поясняю для мудаков:
>>128053
Небесный экватор не наклонён к земному экватору.
>>128055
Солнце движется по зодиакальному кругу по эклиптике.
Аноним 07/10/14 Втр 18:29:34 #38 №128075 
>>128068
>Что происходит с астронавтом, который работает в открытом космосе, когда ему в шлем прилетает какой-нибудь мелкий незарегистрированный неучтённый микрометеороид?
Умирает.
Аноним 07/10/14 Втр 18:37:22 #39 №128079 
>>128068
Зависит от размеров. Микроскопические могут привести к трещине и постепенной разгерметизации, но вполне можно вернуться в шлюз. Даже при полной разгерметизации есть ещё несколько секунд до потери сознания. Или же могут просто немного царапнуть, без каких-либо особенных последствий.
Чуть покрупнее, грамма на три и больше - уже весьма вероятно приведёт к мгновенной смерти из-за ЧМТ, т.к. скорости там в основном гораздо больше, чем у пули, так что несмотря на меньшую, чем у пули, прочность, такой "камешек" запросто пролетит шлем и, даже если разрушится, всё равно будет лететь с большой кучностью, достаточной чтобы пробить череп, мозг и, вероятно, даже проделать с обратной стороны немалое отверстие из-за гидравлического удара. С другой стороны, шлем намного больше головы и тем более мозга, так что более вероятно, что он пролетит мимо головы, либо заденет её по касательной, без смертельной травмы. Но тут уже проблема со стремительной разгерметизацией.
Такого, как в фильме, когда сразу же лопаются глаза, или даже вся голова, или моментально всё замерзает - не будет. Ничего лопаться не будет. Возможно, если будет травма, кровопотеря будет куда более серьёзной. Но внутреннее давление не настолько высокое, чтобы разрывать ткани изнутри. И испарение жидкостей со слизистых тоже не такое быстрое, чтобы привести к мгновенному замораживанию, потому что поверхность слизистых обогревается теплом кровотока быстрее, чем охлаждается из-за испарения. Скорость разгерметизации из-за небольшого отверстия так же не будет настолько высокой, чтобы вызвать травму лёгких, хотя если астронавт попытается задержать дыхание, это всё же может привести к травме.
На людях испытания выживаемости в вакууме, понятное дело, не проводились, но на животных были. И как раз при относительно плавном падении давления они оставались в сознании ещё в течение нескольких секунд, иногда даже десятков секунд.
Если прилетит кирпич, то тут уже всё, пиздец, распидорасило, понятное дело.
Примерно так.
Аноним 07/10/14 Втр 18:44:38 #40 №128082 
>>128079
С астронавтом такой сценарий пока что крайне маловероятен, всё-таки они находятся в космосе в единичном количестве и очень недолго. А вот для спутников уже актуальна проблема защиты от микрометеоритов и на них нередко навешивают решёточки, цель которых - раздробить большие камни в пыль, которая уже сама по себе безвредна, несмотря на большую скорость. Вешать бронелисты, понятное дело, слишком накладно, т.к. каждый грамм на вес золота, да и неэффективно при таких скоростях, т.к. это же не бронебойные снаряды, а камешки из относительно непрочной породы. Так что решётки с определённой конфигурацией достаточны и эффективны. Сам случайно сталкивался с приложениями, выполняющими расчёт столкновения таких тел с решётками для различных конфигураций - скорости тела, его угла, плотности, прочности, паттерна решётки, количества её слоёв, диаметра проволоки и т.п.
Аноним 07/10/14 Втр 18:44:54 #41 №128083 
>>128068
Берем массу говнюлины в 5 грам, на скорости в 20 км\секунду она будет обладать энергией в 100 (сто) килоджоулей.
Пуля 7,62×39 мм - это 2.1 килоджоуль, для сравнения.
Аноним 07/10/14 Втр 18:47:33 #42 №128084 
>>128083
Ну, 20 км/с это ты, конечно, замахнулся. Там скорости примерно 5-15 км/с, нечасто бывает что-то медленнее или быстрее.
Аноним 07/10/14 Втр 18:48:31 #43 №128085 
>>128083
> на скорости в 20 км\секунду
Только на ретроградной орбите. Говнюлины на орбите МКС обычно летают со скоростями сравнимыми со скоростью МКС.
Аноним 07/10/14 Втр 18:52:22 #44 №128086 
Хотя, кстати, скоро уже будет более вероятно, что микрометеоритом будет не каменистый объект, а металлический - космический мусор. А это значит, что такие будут обладать большей прочностью и от них уже никакие решётки не защитят.
Аноним 07/10/14 Втр 19:14:22 #45 №128093 
14126948627330.jpg
Але, спейсаны, это снова я.
Хочу изложить вам один вопрос, который мне как-то давно пришел в голову, но сыскать ответа на него я не сумел. Возможно стоило обратиться даже в наукач, но я в вас верю.

Итак, суть вопроса: зависимость звука от плотности среды.

Как мы все знаем из школьной физики, скорость звука меняется в зависимости от среды. Однако зависимость скорости звука с другими его характеристиками для меня не очевидна. Тут вспоминается другой момент, тоже из школьных времен, а именно, вдыхание в себя шарика с гелием и произнесение всякой хуйни "мультяшным" голосом. Популярное объяснение данного феномена включало в себя проведение корреляции между скоростью распространения звука и его высотой. Казалось бы, при чем тут спейсач?

А спейсач вот при чем. Допустим, есть планета с другим составом атмосферы. А на другой планете она абсолютно точно будет иметь хоть немного, но другой состав. Допустим в данном случае состав отличается значительно и плотность воздуха на ней отличается, скажем, в три раза. Так же предположим, что несмотря на это на планете есть жизнь, в том числе и разумная, которой норм дышать таким воздухом. Жизнь не только разумная, но и развитая настолько, что может совершать межзвездные перелеты и все в таком духе. И вот эта жизнь каким-то образом находит Вояджер-1 и таким же неведомым образом находит способ воспроизвести все записи, хранящиеся на нем, в том числе и записи классической музыки. Вот они прилетают на свою планету и под своим плотным (или разряженным) небом врубают Моцарта в свой инопланетный мафон. Ну и вопрос, собственно: услышат ли они музыку, или искаженный писк/заниженный пердеж и если нет, то какие манипуляции с записью им надо будет провести, чтобы услышать ее так, как слышим мы?
Аноним 07/10/14 Втр 19:49:45 #46 №128111 
>>128093
Причина по которой меняется голос - это плотность гелия, в гелии частота колебаний голосовых связок выше из-за малой массы атомов гелия и малого сопротивления. В элегазе же всё наоборот. Меняются собственные колебания связок, сопротивление газа, скорость истечения из лёгких - всё это.
Когда же мы проигрываем при помощи магнитного динамика запись, то частота колебаний динамика строго фиксирована и зависит только от частоты тока в динамике, но не от плотности среды. Т.о. в гелии запись будет звучать точно также как и в воздухе.
Аноним 07/10/14 Втр 19:51:06 #47 №128112 
>>128093
>услышать ее так, как слышим мы
Это маловероятно. В дополнение к атмосфере, у них даже системы слуха могут быть другие. Может у них нет представления о музыке, о музыкальных инструментах. Может они решат, что это форма языковой связи, а не искусство, которое нужно оценивать человеческим эстетическим чувством.

Тут еще вопросы про их инопланетные проигрыватели встают.

Но я не эксперт, так что пусть еще кто-нибудь ответит.
Аноним 07/10/14 Втр 19:54:27 #48 №128115 
>>128084

>>128085
Алле, болезные. Анон спрашивал про метеоройды, а не про космический мусор. Так что я еще скорость почти по минимому взял.
Аноним 07/10/14 Втр 20:05:54 #49 №128118 
>>128111
Охуенно, анон, я все понял.
Значит искусство будет звучать на всех мирах и не надо переписывать Баха для плотных атмосфер.
Аноним 08/10/14 Срд 00:39:37 #50 №128142 
>>128118
Ага, главное, что бы ухи у них работали в том же диапазоне частот.
Аноним 08/10/14 Срд 05:49:34 #51 №128166 
>>128115
>>128075
>>128083
>>128082
>>128079
Но ведь никто ещё не убился при выходе в открытый космос об микрометеороиды.

Космический мусор, учтённые большие метеориты, кирпичи, прочая херь, траектории которых рассчитаны заранее не в счёт. Ясно что убьёт нахуй. И ясен Хуй что астронавт не полезет под этот обстрел.

Я спрашивал про мелкие объекты в поперечнике буквально сантиметры или даже меньше, то, что нельзя как-то засечь с земли. Какая от этого защита и как космонавт переживает попадание пылинок в него. Это как удар по ебалу, как укол иголкой или как-то иначе ощущается телом? Или настолько назначительно что вообще не почувствуешь.

И да, я понимаю что объекты мелкие и неплотные/непрочные, но если даже такая хуйнюшка летит из-за пределов сферы влияния Земли, скажем, из пояса астероидов, а то и из пояса Койпера, то во-первых её никак не заметить, а во-вторых, скорость при самых неблагоприятных условиях достигнет 50-70 км/с, а это даже при относительно мелких массах ощутимые такие энергии.
Аноним 08/10/14 Срд 09:27:08 #52 №128197 
14127460280990.jpg
Как-то видел сайт, где опровергается опровержение высадки американцев на луну. Не могу найти, кто-нить поможет? Или может фильм есть какой.
Аноним 08/10/14 Срд 09:47:10 #53 №128199 
>>128197
http://lurkmore.to/Луносрач
На почитай, может будешь , ньюфаг, лучше понимать нас, олдфагов.
Аноним 08/10/14 Срд 10:07:44 #54 №128202 
>>128166
Ну так сравни масштабы. Это все равно, что играть в морской бой со стандартными правилами на поле 1000000х1000000. Нескольких тысяч лет может и хватит, чтобы завершить партию, но 50-ти лет не хватит даже чтобы "попасть" однажды. Также есть теория, что в системе планета-крупный спутник большую часть летящей извне мелочи таки будет собирать спутник.
Аноним 08/10/14 Срд 10:41:51 #55 №128203 
14127505115560.jpg
Спейсач, помоги с одной теорией. Есть 3 допущения, которые вытекают одно из другого. Как их опровергнуть?

1) В будущем будут разработаны расшифрованы сигналы мозга и разработаны интерфейсы мозг-компьютер, компьютер-мозг. Что-то наподобие флеш разъема за ухом.

2) В будущем люди смогут объединять свои мозги в сеть мозг-компьютер-мозг. То есть два или более людей смогут напрямую обмениваться мыслями, мозговыми сигналами, образами, идеями.
Люди объединенные в такую сеть получают преимущество над индивидуалистами за счет своей скоростной коммуникации и координации действий.
Все больше людей признают такой метод объединения и активно пропагандируют "открытость". Тот кто не присоединяется к мысле-сети, кому есть что скрывать от большинства, тот объявляется террористом и насильственно присоединяется к "открытости".
Со временем люди теряют представление о Я и его заменяет такое понятие как Мы.

3) Шансы встретить в космосе разумных индивидуалистов стремятся к нулю. Все цивилизации в космосе являются такими общими Мы, состоящими из материально давно мертвых пришельцев.
Аноним 08/10/14 Срд 10:53:07 #56 №128204 
>>128199
О блять боже, олдфаг, язык лучше в жопу себе засунь, петуш блядь.
Аноним 08/10/14 Срд 11:44:48 #57 №128218 
>>128203
1) Или нет. Или без разъёма. При этом есть вероятность, что расшифровать вообще всё, не получиться из-за особенностей мышления конкретного человека. и.т.д

2) Или не смогут. Или будут обмениваться только строго определённым набором данных.
Если они и получат преимущество, то только внутри своей группы. При этом не факт, что это вообще даст что-то кроме лицокнига версии 2.0. Конечно, если ты не сводишь всё к командному спорту. Сами по себе коммуникация и координация действий ещё ничего не дают. Как тебе нейронная сеть состоящая из ТП, школоты, поехавших, задротов, кришнаитов, дугинистов и.т.п. И почему ты не учитываешь, что может быть несколько конкурирующих между собой сетей? А теперь представь себе, что этот коллективный филиал психушки и потреблятской тусы начнёт всех убеждать, что надо стать одним из них… Тут ещё вопрос, кто кого отключит/подключит.

3) А тебе есть разница кто перед тобой? Коллективное Я целой цивилизации загруженное на сервер и отправленное в космос или индивидуальное Я загруженное на сервер и отправленное в космос. Всё равно общение будет проходить в формате диалогу между двумя сущностями. Просто не вижу принципиальной разницы, при условии, что технологии и способы их использования примерно одинаковы.
Аноним 08/10/14 Срд 12:04:54 #58 №128226 
>>128166
Вероятность, что эта хуйня прилетит из дальних рубежей, чтобы попасть по отдельно взятому астронавту... в общем, вокруг Земли на порядки больше мусора.
Аноним 08/10/14 Срд 15:15:46 #59 №128263 
>>128166
>Но ведь никто ещё не убился при выходе в открытый космос об микрометеороиды.
Вероятность крайне мала такого события, вот никто и не убился.
>Я спрашивал про мелкие объекты в поперечнике буквально сантиметры или даже меньше
>Какая от этого защита
Я тебе написал что будет >>128083. Мгновенная смерть, защиты от таких подарков нет никакой.
Аноним 08/10/14 Срд 15:30:43 #60 №128266 
>>128197
>Как-то видел сайт, где опровергается опровержение высадки американцев на луну
Петушиные визги никому не интересны. Ты еще попросил бы гороскоп тебе составить. Если любишь говна поесть, то ищи его сам.
>Что-то наподобие флеш разъема за ухом.
Скорее это будут беспроводные системы, которые в зачаточном состоянии есть уже сейчас.
Например япохи продают кошачьи ушки, которые меняют свое положение, в зависимости от эмоций носителя. Еще есть всякие устройства для инвалидов, много чего есть уже сейчас, но пока не очень все это работает, однако прогресс в этом направлении вполне себе идет.
>Люди объединенные в такую сеть получают преимущество над индивидуалистами
Такое-то О-сознание, как в ждалкере.
Вполне возможно, что будут такие вещи, но под конкретные задачи, что-то типа нынешних конференций по скайпу.
> Тот кто не присоединяется к мысле-сети, кому есть что скрывать от большинства, тот объявляется террористом и насильственно присоединяется к "открытости".
А вот это уже раса боргов из стартрека, там все соединены в одно целое, не думаю, что это тоже получится, в итоге человечество соснет, ибо весь прогресс делают отдельные личности, базируясь на фундаменте общих идей, социума. Без индивидуализма человечество не выживет, ибо склонно к иерархическим парадигмам.
>Шансы встретить в космосе разумных индивидуалистов стремятся к нулю.
Это такое же утверждение, как то, что мы будем встречать сложных разумных существ, состоящих из одной индивидуальности.
Общим Мы, может восприниматься и человечество. Прилетают пришельцы, тут 7 милиардов более-менее разумных идиотов, а встречать и устанавливать контакт придет группа из 3.5 анонимусов, которые будут вещать за всех. И это стандартное вспомни, характерное для людей: "От лица компании" "От лица народа" "От лица сообщества" Всегда пиздит кто-то один, выбранный из толпы, но от лица всех, хотя этих всех никто не спрашивал.




Аноним 08/10/14 Срд 15:31:09 #61 №128267 
>>128203
>>128266
sageАноним 08/10/14 Срд 15:36:03 #62 №128270 
>>128266
>Например япохи продают кошачьи ушки, которые меняют свое положение, в зависимости от эмоций носителя.
Хотеть! Сцылки на Амозонь!
Аноним 08/10/14 Срд 15:49:33 #63 №128272 
>>128270
Гугли Necomimi
Аноним 08/10/14 Срд 17:53:17 #64 №128291 
>>128266
> Скорее это будут беспроводные системы, которые в зачаточном состоянии есть уже сейчас.
Нет, крайне сомнительно, чтобы эта технология могла обеспечить сколько-нибудь приемлемую широту функционала.
Эти приборы улавливают только поверхностные потенциалы, а это только кора головного мозга. К тому же, они улавливают активность сразу очень больших участков мозга и неспособны (и скорее всего не будут способны) различить работу отдельных групп нейронов или тем более отдельных нейронов. А это скорее всего будет необходимостью для полного декодирования мыслительной активности.
Аноним 08/10/14 Срд 19:57:36 #65 №128301 
>>128291
>и скорее всего не будут способны
А вот это еще посмотрим. При более детальном изучении, вполне возможно, найдется соответствующая технология, позволяющая более дискретно разделять импульсы. В этом нет никакой фантастики, нужны сотни науки и инженерии. По дебилятору видел устройство, которое позволяет поднимать и крутить виртуальный кубик силой мысли, а так же катать инвалидную коляску и управлять ею. А это уже намного сложнее, чем просто реагировать на общий "фон" импульсов.
Аноним 08/10/14 Срд 20:28:45 #66 №128304 
>>128301
Ненамного. И точность таких манипуляций оставляет желать лучшего, иначе бы паралитики уже давно играли бы в футбол - у них-то времени для выработки навыка работы с таким интерфейсом более чем достаточно.
И я знаю такие устройства, даже думал себе прикупить такой, мол, было бы классно играть в игры, не прикасаясь к клаве и мышке. Но потом покопался, почитал отзывы, посмотрел как это в реале выглядит, и понял, что до сколько-нибудь приемлемой точности и реакции это не довести. В принципе, можно научится распознавать несколько десятков отдельных областей, "очищать" их от шума соседних областей, тем самым получить несколько десятков степеней свободы у такого контроллера, но больше того... едва ли. Тем более тут и речи быть не может о том, чтобы различать группы нейронов.
Понимаешь, анон, это как пытаться распознать что находится за метрами плотного тумана, стоя на месте. Свет оттуда к нам доходит, можно даже различить из какого направления идёт какой цвет, но никакой наукой невозможно собрать этот свет в чёткую картинку, потому что туман - система хаотичная и проходящий свет преломляется и отражается бесчётное количество раз, что делает вычисление исходного направления хода луча нереальным даже с технологиями будущего, потому что даже имея теорию всего, даже зная характеристики каждого лептона, бозона и чего там ещё есть, потребуется сжечь звёзды, если не галактики для вычисления исходных характеристик света и построения картины того, что находится за туманом.
В данном случае кора, череп и скальп - это те самые метры тумана.
Да и в любом случае, это не тот способ, каким работает мозг. Использовать ЭЭГ для управления чем-либо - это как забивать гвозди микроскопом. Можно наловчится активировать отдельные области коры, но вот как это скажется на самих нейронах коры и их связях с остальными отделами мозга и, главное, на самой работе мозга?
Аноним 08/10/14 Срд 20:39:12 #67 №128307 
Вот, кстати, когда-то в процессе обоснуя для комикса, пришла одна идея - нейроинтерфейсы обычно представляют себе, как некий механический контакт, кучу микроскопических проводков, соединённых с нейронами. Но это ведь очень сложно, затратно и долго. А платы с микроскопическими иголочками, входящими в контакт с нейронами, неточны и неприменимы в больших масштабах. Остаются нанороботы, но до них ещё далеко, да и ими тоже надо управлять, что опять же сложно в таких масштабах. Не лучше ли будет использовать структуры, подобные самим нейронам? Например, генетически модифицированный гриб, который бы прорастал в мозг, создавал бы односторонние контакты с нейронами (односторонние - чтобы не "замыкало") и уводил бы это всё в свой центр, который уже находится в месте поудобнее - хоть на затылке, как в матрице, хоть где. Ну или же не гриб, а сам нейрон, с изрядно переписанным геномом.
Дискасс?
Аноним 08/10/14 Срд 20:49:11 #68 №128310 
>>128307
Тут есть одна интересная лекция про киборгизацию и интерфейсы мозг-компьютер. Там как раз обсуждается сегодняшний уровень технологии и перспективы развития, без фантастики, на примере реальных разработок.

http://www.youtube.com/watch?v=6LGWBl1qRsU
Аноним 08/10/14 Срд 20:53:47 #69 №128311 
>>128307
Сложность создания "ГМО-гриба-интерфейса" значительно превосходит сложность создания "интерфейс-провода через нос". Когда ты сможешь делать такие "грибы" они уже не будут востребованы.
Аноним 08/10/14 Срд 21:02:33 #70 №128312 
>>128311
Сложность создания технологии - возможно. Но вот сложность конечного применения - едва ли, по части внедрения это напротив должно быть несопоставимо дешевле.
Аноним 08/10/14 Срд 21:14:28 #71 №128314 
>>128312
Дело не в цене. "Гриб" устареет ещё до момента появления готового продукта. Грубое сравнение. Сделать супер-пупер парусник, работа над которым началась в бронзовом веке, когда все используют ракетные крейсера.
Аноним 08/10/14 Срд 22:26:31 #72 №128322 
Во сколько сегодня звездопад по Москве? И сколько он будет примерно?
Аноним 08/10/14 Срд 22:41:00 #73 №128323 
>>128322
>Во сколько сегодня звездопад по Москве?
Смешно. "Звездопад" Дракониды где-то 8-10 октября, по московскому времени. Они не приходят по часам. Этой ночью ожидается максимум. Дракониды тем более, очень непредсказуемый поток, так что можешь начинать смотреть в небо прямо сейчас
>>128304
>Ненамного. И точность таких манипуляций оставляет желать лучшего, иначе бы паралитики уже давно играли бы в футбол
А ты забавный. Я тебе ломающие новости принес.
http://www.youtube.com/watch?v=P2BAnADc_js
Бионический глаз уже сейчас способен частично вернуть зрение.
http://rspb.royalsocietypublishing.org/content/280/1757/20130077.full
Алсо, уже есть прототипы нейроимплантов, нивелирующих симптомы болезни Паркинсона.
Аноним 09/10/14 Чтв 00:48:45 #74 №128333 
14128013251130.jpg
>>128304
> ыло бы классно играть в игры, не прикасаясь к клаве и мышке
Аноним 09/10/14 Чтв 03:40:46 #75 №128336 
>>128323
Ты не в тему.
Глаз - имплант, здесь же речь идёт о неинвазивном интерфейсе на базе ЭЭГ мозга.
Протезы могут быть с имплантами, могут быть на базе ЭЭГ, но в последнем случае они ограничены только одной конечностью и никак не опровергают то, о чём я говорю.
Аноним 09/10/14 Чтв 05:04:09 #76 №128339 
>>128336
>Все-таки проблема мне видится в недостаточной изученности процессов мозга, плюс несовершенство технологий. Со временем наверняка найдут способ отфильтровать "левые" шумы и вычленить мыслительную составляющую, например образы, для начала. Процесс сложный, но все-таки мозг - это не магия.

sageАноним 09/10/14 Чтв 07:31:55 #77 №128344 
>>128339
Вернись в обо/сцай, дегенерат.
Аноним 09/10/14 Чтв 13:33:56 #78 №128363 
Какова максимальная цветовая температура сверхновой? Или у неё не тепловой спектр?
Аноним 09/10/14 Чтв 13:47:17 #79 №128364 
>>128363
За 10 сут до максимума цветовая температура сверхновой I типа составляет примерно 50 тыс. К, а вблизи максимума 15 тыс. К. После максимума блеска уменьшение цветовой температуры происходит еще примерно 30 сут, а затем при темп-ре ок. 5 тыс. К прекращается.
Аноним 09/10/14 Чтв 14:11:11 #80 №128379 
>>128364
> а вблизи максимума 15 тыс. К
в смысле 150? ну ясненько(((
Аноним 09/10/14 Чтв 15:40:25 #81 №128386 
>>128379
В смысле 15 тыс.
Аноним 09/10/14 Чтв 16:10:03 #82 №128390 
>>128272
Охуенный совет, браток. Ты как в каком-то буржуйском фильме, где главный герой загуглил "computer" и "File" и на первой странице получил то, что нужно нет, ему не была нужна англовики
Аноним 09/10/14 Чтв 16:15:33 #83 №128391 
>>128307
>>128266
Гугланите оптогенетику, например. Там суть в том, что трансгенным мышам в мембрану нейрона захуярили специальные штуки, которые обеспечивают возбуждение под действием фотонов, а потом просверлили башку и провели оптоволокно куда надо. Где-то на ютубе есть даже видео, в котором мышь с компьютера заставляют сворачивать налево или направо, как захотят.
До трансгенных человеков нам ещё далеко, но в принципе идея искусственно делать нейроны чувствительными к какому-нибудь легко модулируемому сигналу хороша.
Аноним 09/10/14 Чтв 16:44:03 #84 №128396 
>>128390
Он имел в виду Neurowear, очевидно же.
Аноним 09/10/14 Чтв 22:28:34 #85 №128468 
>>128379
Смайлофаг петросянит. Мерзкое зрелище.
>>128390
На здоровье.
>Он имел в виду Neurowear
Я имел ввиду, то что написал. Продукт называется Necomimi. Neurowear - это название конторы, которая их разработала.
Аноним 10/10/14 Птн 02:52:18 #86 №128554 
>>128386
>>128468
Какое блять петросянство? Как это вблизи максимума меньше? А в максимуме сколько тогда?
Аноним 10/10/14 Птн 03:23:35 #87 №128555 
>>128554
Ты не оговорился ли? Анон написал, что вблизи максимума больше, чем в максимуме, ты, очевидно, решил, что там опечатка и что правильно 150K.
Короче, я сам не в курсе, но могу предположить, что температура падает, потому что максимум блеска наступает существенно позже взрыва, когда вещество уже прилично разлетелось и остыло, становясь тем самым более прозрачным и пропуская наружу больше света. Поправьте, если ошибаюсь.
Аноним 10/10/14 Птн 09:14:29 #88 №128565 
>>128554
>Как это вблизи максимума меньше?
Там все верно написано. Цветовая температура не совпадает с эффективной температурой.


Аноним 10/10/14 Птн 11:47:19 #89 №128594 
Можно ли вылететь за марс ювелирно сворачивая у нашей плонети?

Имеется в виду, сворачивая в системе земля-луна?
Аноним 10/10/14 Птн 12:14:47 #90 №128597 
>>128594
>Можно ли вылететь за марс ювелирно сворачивая у нашей плонети?

Зависит от траектории, скорости и положения планет. Гравитационную пращу юзают с начала космонавтики.
Аноним 10/10/14 Птн 12:25:55 #91 №128602 
14129295557580.jpg
14129295557631.jpg
14129295557652.jpg
14129295557673.jpg
>>128468
>Продукт называется Necomimi.
Ебать-копать, какое оригинальное название, как легко по нему гуглить.
Аноним 10/10/14 Птн 12:42:19 #92 №128605 
>>128602
Ну что такое? Айфон тоже охуеть оригинальное название, ничего, все привыкли.
Аноним 10/10/14 Птн 15:50:19 #93 №128637 
14129418193260.jpg
Так всё-таки, высаживались мериканцы на Луне или нет? После анализа всех доводов, кажется что остались только косьвенные улики. Самый значимый, наверно, это отсутствие современных чётких снимков Луны. Снимков не с говно-LRO, а с мощных телескопов. Киньте в меня пруфом, если есть.
Аноним 10/10/14 Птн 16:18:22 #94 №128639 
>>128637
Ты вообще в курсе, как работают телескопы и почему даже Хаббл не сможет сделать фотографию лучше, чем ЛРО, мамки детектив-конспиролог?
Аноним 10/10/14 Птн 16:21:22 #95 №128640 
>>128594
По Межпланетной транспортной сети можно улететь даже к Аллаху, достаточно выбраться на НОО, заебешься разве что.
Аноним 10/10/14 Птн 16:27:22 #96 №128641 
>>128637
>Так всё-таки, высаживались мериканцы на Луне или нет?
Платина.

>После анализа всех доводов, кажется что остались только косьвенные улики.
Имбецил.

>0.5 m/pix
>говно-LRO
Вообще пушка.

>а с мощных телескопов
Школоло-физик в треде, который хочет разглядеть объект размером в 0.000019 угловых секунд в телескоп.

Теперь съебал в ZOG, быстро, решительно. Здесь паранойю не лечат.
Аноним 10/10/14 Птн 16:31:59 #97 №128642 
>>128637
Идешь сюда:
http://www.skeptik.net/conspir/moonhoax.htm
Читаешь все подряд 10 раз.
Если у тебя после этого остаются сомнения, то значит ты дурак от природы.
Аноним 10/10/14 Птн 16:49:20 #98 №128644 
14129453605840.jpg
>>128641
>>128639
Ясно, значит пруфов нету. На луне никого не было!
Аноним 10/10/14 Птн 16:56:44 #99 №128646 
>>128642
я это читал

Про телескопы там быстро проскочили, темку-то замяли. А есть телескопы поболе чем там пишут. В Чили 4 зеркала по 8 м, можно не то что ровер разлядеть, следы от ботинок. https://ru.wikipedia.org/wiki/Very_Large_Telescope Вот только им всем строго-настрого запрещено делать снимки луны.
Аноним 10/10/14 Птн 17:05:19 #100 №128648 
>>128646
>В Чили 4 зеркала по 8 м, можно не то что ровер разлядеть, следы от ботинок.
NOPE
>Вот только им всем строго-настрого запрещено делать снимки луны.
В /zog/
Аноним 10/10/14 Птн 17:32:35 #101 №128652 
>>128646
Не, ну давай уж, посчитай нам, снимки какого разрешения может делать этот телескоп на поверхности Луны. Конкретно.
Аноним 10/10/14 Птн 17:54:18 #102 №128654 
>>128652
Далеко ходить не надо. Вот сайт обсерватории:

http://www.eso.org/public/teles-instr/vlt/

Там написано, что разрешающая способность, при объединении телескопов в один массив, даст разрешение в несколько милисекунд дуги, это как разрешить свет фар автомобиля на два источника на расстоянии до Луны.
Таким образом ни о каких разглядываний подробностей посадки Аполлонов речи и близко не идет.
Аноним 10/10/14 Птн 18:21:55 #103 №128661 
>>128654
>несколько милисекунд дуги
Угловой диаметр Луны - 1 800 секунд дуги, диаметр, 3500 км. Т.о. одна миллисекунда - это 2 метра. Т.о. разрешение телескопа VLT на поверхности Луны составляет порядка 10 метров.

мимокрок
Аноним 10/10/14 Птн 18:57:57 #104 №128670 
>>128661
>разрешение порядка 10 метров.

Диаметр посадочного модуля Аполлона - 4,3 м.
ВЛТ пролетает мимо.
Аноним 10/10/14 Птн 19:31:22 #105 №128693 
Господа, да это же либо недотрололо-школьник, либо очередной школьник имбецил. Ну, может, конечно, это и просто имбецил от природы, но в любом случае, нахуй ему что-то отвечать? Захочет - сам найдёт. С такими предъявами, когда он себя сразу позиционирует в имбецила, шансов на излечение практически нет.
Аноним 10/10/14 Птн 19:39:15 #106 №128698 
>>128694
>Ну, может, конечно, это и просто имбецил от природы
Это тред тупых вопросов, здесь можно задать самый тупой вопрос. Считаю, что исходить нужно из этого.
Если поциент продолжает тупить, то тогда уже ясно, что это трололо.
Алсо, мы тут неплохо развлеклись. Я вот узнал, что ВЛТ способен собираться в единый массив, с очень некислым разрешением. Нарыл, какой будет угловой размер воткнутого флага на Луне, анон выше рассчитал разрешение массива ВЛТ в метрах.
Ищи выгоду, короче.
Аноним 10/10/14 Птн 19:39:34 #107 №128699 
>>128698
>>128693
Промазал
Аноним 10/10/14 Птн 19:52:00 #108 №128703 
>>128698
Я тебе больше скажу - все цифровые телескопы Земли теоретически можно объединить в один массив (и более того - многие крупные телескопы планеты нередко объединяют) и получить охуенное разрешение при неплохой чувствительности. Это примерно как собрать телескоп размером с саму Землю, только с чувствительностью, на порядки меньшей, чем если бы это был один сплошной рефлектор соответствующих размеров.
Аноним 10/10/14 Птн 19:55:00 #109 №128704 
>>128703
>>128698
Ну и, кстати, не только телескопы Земли можно объединять, но и орбитальные телескопы тоже, как например наш Радиоастрон.
Тут вся загвоздка в том, чтобы согласовать их данные по времени с огромной точностью, а это означает учёт даже релятивистских эффектов. И чем дальше расставлены телескопы, тем большая точность согласования требуется.
Аноним 10/10/14 Птн 20:13:31 #110 №128705 
>>128703
>Я тебе больше скажу
Спасибо за ликбез, я про это знаю, не знал, что в ВЛТ эта фича заложена изначально.
>все цифровые телескопы
Лол, что за цифровые телескопы?

>и более того - многие крупные телескопы планеты нередко объединяют
Объединяют обычно радиотелескопы в сверхдлинные базы. Про оптические в едином массиве я вот впервые узнал.
>как например наш Радиоастрон
Наш Радиоастрон вообще-то один и не работает, он именно запущен, что бы иметь сверхдинную базу, в сети с наземными телескопами. Это его изначальное предназначение.
Аноним 10/10/14 Птн 20:19:57 #111 №128706 
>>128705
Цифровые - в том смысле, что их вообще все в теории можно объединить. Только кому это нафиг надо?
А чем телескопы, работающие в оптическом диапазоне, должны так уж отличаться от радиотелескопов?
Аноним 10/10/14 Птн 20:58:26 #112 №128711 
>>128706
>А чем телескопы, работающие в оптическом диапазоне, должны так уж отличаться от радиотелескопов?
Приемником, лол.
Аноним 11/10/14 Суб 09:49:52 #113 №128801 
http://www.stefanom.org/spc/# играл кто? Как фриплей сделать, чтоб игра не стопалась после вылета, - только задонатив?
Аноним 11/10/14 Суб 10:18:55 #114 №128805 
>>128706
>А чем телескопы, работающие в оптическом диапазоне, должны так уж отличаться от радиотелескопов?
1) Длиной волны. При работе в режиме интерферометра точность совмещения сигналов пропорциональна частоте, т. е. радиотелескопы метрового диапазона в миллион раз проще объединить, нежели оптические. С другой стороны, разрешение телескопов само по себе зависит от длины волны, и оптический телескоп при одинаковом с метровым радиотелескопом диаметром зеркала имеет в миллион раз большее разрешение.
2) Дрожание атмосферы, которое необходимо устранять.
Т.о. объединить оптические телескопы на расстоянии 100 метров друг от друга ещё можно, но на расстоянии нескольких километров нереально в ближайшее время.
Аноним 11/10/14 Суб 10:29:31 #115 №128806 
>>128805
3) Кроме того, при наблюдении в режиме интерферометра платой за высокое разрешение является катастрофическое снижение сигнала. Все интерферометры работают в режиме сверхдолгих наблюдений с накоплением сигнала. Это не проблема для радиотелескопов, которые работают при любой погоде, это не проблема для космических обсерваторий, например Хаббла, которые могут обозревать участок неба годами. Но это проблема для наземных телескопов, которые вынуждены обозревать участок неба учитывая все атмосферные явления. Не факт, что годы наблюдений можно отфильтровать от шумов.
4) Кроме того, небо на Земле сильно засвечено для таких наблюдений.
Аноним 11/10/14 Суб 10:31:38 #116 №128807 
>>128806
А если интерферометры на луне разместить, норм будет?
Аноним 11/10/14 Суб 10:45:51 #117 №128809 
>>128807
Гораздо проще на орбите (придется долго-долго собирать сегменты). Но один хуй, остальное никуда не денется.
Аноним 11/10/14 Суб 11:16:30 #118 №128814 
>>128809
На орбите сложнее отслеживать расстояние между сегментами. К примеру у Радиоастрона серьёзные сложности с синхронизацией на такой длинной базе. В этом плане как раз Луна выгоднее.
Аноним 11/10/14 Суб 17:59:03 #119 №128868 
>>127536
Анонасы, поясните мне по хардкору, что такое солнечно-синхронная орбита. На википедии что-то нихуя не понял.
Аноним 11/10/14 Суб 19:49:46 #120 №128911 
>>128868
Это орбита, на которой спутник проходит одну и ту же точку на поверхности Земли в одно и то же местное солнечное время. Период вращения Земли 23h56', тогда как сутки длятся 24 часа. Следовательно для того, чтобы приходить в одну и ту же точку над поверхностью Земли в одно и то же солнечное время орбита спутника должна прецессировать вокруг Земли со скоростью 1 градус в день. Орбиты спутников всегда прецессируют вокруг Земли из-за её экватриального вздутия, при том орбиты с разным наклоном прецессируют по-разному. ССО это орбита с наклоном 94 градуса, т.е. полярная (вертикальная) и немного ретроградная (направленная против вращения Земли), и только такая орбита прецессирует вслед за Солнцем со скоростью 1 градус в день.
Аноним 11/10/14 Суб 19:59:26 #121 №128914 
>>128911
Ах да, период обращения спутника по ССО кратен 24 часам, соответственно он может по 15 раз в день пересекать экватор в одно и то же местное время (например в 12 часов дня), так как плоскость его орбиты относительно направления Земля-Солнце неподвижна. Т.е. пока он летит по дневной стороне Земли под ним всегда (если не учитывать небольшой наклон в 4 градуса) местное солнечное время 12 часов дня на всём полувитке, потому что он летит в той же плоскости, где лежит Солнце.
Если повернуть орбиту относительно линии Земля-Солнце, то местное солнечное время под спутником можно установить хоть 10 часов дня, хоть 15.
Аноним 12/10/14 Вск 11:55:54 #122 №129044 
Могжет ли у планеты быть несколько колец под разным наклоном орбиты?
Аноним 12/10/14 Вск 12:35:50 #123 №129058 
>>129044
Кольца устойчивы только в плоскости экватора, вроде.
Аноним 12/10/14 Вск 19:37:26 #124 №129160 
>>129044
Нед. Только в экваториальной плоскости.
Аноним 12/10/14 Вск 22:00:13 #125 №129171 
>>129058
>>129160
А если по этой орбите крутится искусственный спутник, который оставляет за собой след из неких частиц, постоянно пополняя нестандартное кольцо?
Аноним 12/10/14 Вск 22:05:18 #126 №129172 
>>129171
Частицы, выпавшие со спутника будут лететь с ним рядом параллельным курсом.
Аноним 12/10/14 Вск 22:05:46 #127 №129173 
>>129171
Частицы, выпавшие со спутника будут лететь с ним рядом параллельным курсом.
Аноним 12/10/14 Вск 22:30:44 #128 №129180 
>>129172
>>129173
Может они ещё и параллельным курсом будут лететь?!
Аноним 12/10/14 Вск 23:11:55 #129 №129192 
Космоны, а онлаен телескопы реальные есть где нибудь? Ну тип, зашел на страничку, покрутил телескопом, где нибудь, в Чили, посмотрел няшноты.
Аноним 13/10/14 Пнд 01:26:07 #130 №129214 
>>129192
Есть, можно делать фоточки с удаленного телескопа за прайс. Куча есть таких обсерваторий.
На Хабре пост был подробный. http://habrahabr.ru/post/200640/
Аноним 13/10/14 Пнд 09:04:12 #131 №129249 
>>128641
>>128639
>тред тупых вопросов
>бля чет у тебя вопросы тупые
Аноним 13/10/14 Пнд 11:50:45 #132 №129250 
>>129249
Там очевидное троллоло.
Аноним 13/10/14 Пнд 12:48:26 #133 №129257 
>>129250
Автор утверждает, что проанализировал все доводы, а сам вообще не рубит. Таких надо макать в говно.
Аноним 14/10/14 Втр 01:33:40 #134 №129326 
Спейсач, зашел на днях в sci. Господи, какие же там все ебанутые и тупые. Какое счастье, что мой компас ламповый и хороший. Добра всем.
Аноним 14/10/14 Втр 01:45:24 #135 №129327 
>>129326
Ты хоть помылся, прежде, чем сюда вернуться?
Аноним 14/10/14 Втр 07:24:52 #136 №129343 
14132570921650.jpg
На Луне - луноход. На Марсе - марсоход.

Как будет называться меркурианский, венерианский или церерианский планетоход? Меркуриход, венероход, церероход?
В английском все просто - Mercurian rover или Venus rover, а в русском как сказать красиво?
Аноним 14/10/14 Втр 09:49:42 #137 №129360 
>>129343
Планетоход - общепринятое название в спец литературе, эквивалент "ровера".
Аноним 14/10/14 Втр 12:59:47 #138 №129363 
>>129343
Страшно подумать, как будет называться планетоход на Земле.
Аноним 14/10/14 Втр 18:00:06 #139 №129396 
>>129343
Да, примерно так. А так же не забывай про примеркурение, привенерение и прицецерение.
Аноним 14/10/14 Втр 18:59:07 #140 №129414 
14132987473270.jpg
По какому принципу распределяют ступени на ракетах, по геометрической прогрессии, или может формула хитрожопая есть? То есть, например, надо 4500 м/с, штаб выйти на орбиту Кербина, сколько дельты вкладывать в первую ступень, вторую, и т.д. Вопрос не столько в том, как эффективнее сделать, хотя и в этом тоже, а в том, чем руководствуются при конструировании.
Аноним 14/10/14 Втр 19:05:29 #141 №129419 
>>129414
Не знаю точно, но рискну предположить, что какого-то прямо строгого правила, определяющего количество ступеней, не существует. Чем больше ступеней - тем лучше, при условии их пропорциональности. Ведь сам смысл ступеней из нескольких разгонных блоков в том, чтобы на каждом следующем этапе иметь как можно меньше массы, т.к. это повышает эффективность. И идеальное значение стремится к бесконечности. Но чем больше ступеней, тем сложнее конструкция и тем больше риски.
Аноним 14/10/14 Втр 22:12:38 #142 №129424 
>>129419
И то - это если у нас невесомые двигатели.
Аноним 15/10/14 Срд 10:31:24 #143 №129442 
>>129414
Всегда стараются делать поменьше ступеней, ибо каждое лишнее разделение, включая обтекатели, сильно снижает надежность. Одну ступень сделать не позволяет эффективность (удельный импульс существующих двигателей), но зачастую делают в ущерб эффективности в пользу простоте (Falcon 9 например).

Про распределение массы ступеней - есть куча условий - можем сделать двигатель на столько-то тяги такой-то массой, баки столько-то, скорость разделения в атмосфере не выше стольких-то, температурный режим такой-то, ступень надо ронять там-то, и т.п. В соответствии с ними разными способами ищется оптимальное решение по выходным параметрам, включая распределение масс, траекторию полета и т.п.
Аноним 15/10/14 Срд 12:45:53 #144 №129452 
14133627531560.png
>>129442
Находят минимумы у этой функции.
Аноним 17/10/14 Птн 23:03:55 #145 №129737 
У Ангары какое бортовое электропитание? На батарейках, как у протона или турбогенератор? А у вражин как обстоят дела с питанием систем вражеских ракет?
Аноним 18/10/14 Суб 01:51:30 #146 №129753 
>>127536
Космач, недавно я внезапно задался вопросом. А зачем нам другие планеты? Нет, ну серьезно. Орбита - понятно, спутники, которые упрощают повседневную(и не только) жизнь. А планеты? Я, считаю, что это просто охуительно, что человек обязан колонизировать солнечную систему. Но вот глазами стороннего человека или же правительства? Это просто огромные деньги нужны, которые не отобьются. Можно рассчитывать на внедрение технологий, полученных при разработке, в жизнь каждого человека - только это в перспективахвспомнить, хотя бы те же ssd, которые первыми использовались как раз в аэроспейсе, а пришли к нам относительно недавно Ведь прямого профита космос не принесет? Полезные ископаемые есть на Земле, что-то, чего нет здесь, нам не нужно с Луны или Марса. Так в чем профит будет? И будет ли он вообще?
Про Гелий-3 и т.п. можно не писать, мы еще далеко от термоядерного синтеза.

Повторюсь, я считаю, что человечеству необходимо это сделать, однако задаю вопрос как бы со стороны.

http://vimeo.com/106216724
Аноним 18/10/14 Суб 02:54:41 #147 №129763 
>>129753
Ну во первых до полноценной колонизации планет мы ещё относительно далеки. Так, прилететь и исследовательскую станцию поставить можем. Но эти станции крайне важный шаг для движения дальше.
А космос нужен для ресурсов. Тот же гелий-3 мы гораздо ближе к термояду чем ты думаешь . Кроме того чтобы налутать всякой платины, палладия и железа на земле надо хорошенько поебаться с малым выхлопом. В том же поясе астероидов можно найти самородочки на несколько мегатонн почти чистого материала. Да и газа на том же Титане куда больше чем у Газпрома.
Ну и лишних людишек с планеты куда-нибудь сбагрить не помешает, а то при численности населения в овер 10 гигачеловек к 20 году может тупо не хватить жратвы, а пилить гидропонику в сахаре ненамного дешевле и проще чем лунную базу.
Да и вообще человечество как дитё малое - чем бы ни занималось лишь бы не рукоблудило, а то без какой-то глобальной объединяющей цели вроде яблонь на марсе можем и до ТМВ хуи допинать.

Аноним 18/10/14 Суб 03:22:33 #148 №129768 
Аноны, вот на земле есть приливные электростанции, понятно что луна постепенно удаляется от земли, но если представить шарообразную луну в вакууме (лол), которая все время будет находится на одном расстоянии можно ли будет считать приливные электростанции сорт оф вечным двигателем ?
Аноним 18/10/14 Суб 03:33:22 #149 №129769 
>>129753
Это как бы и не вопрос. Какой-либо колонизации Солнечной системы мы не видим и не увидим потому, что:
1. в этой системе нам известен ровно один уголок, где человек может прожить без скафандра дольше пяти минут - это планета Земля;
2. никто пока не предложил экономически обоснованную причину разворачивать какую либо промышленность или добычу где-либо в космосе. С существующими средствами доставки, даже если астрономы завтра найдут астероид из чистого палладия, добыча и доставка этого палладия на Землю обойдётся дороже, чем стоит местный палладий. Особенно, учитывая, что цена на этот металл сразу же упадёт;
3. договор об освоении космоса похоронил все политические причины для государств разворачивать какие-либо военные базы или поселения с целью "застолбить" потенциально полезную собственность.
Собственно всё. Пилотируемые полёты времён Холодной войны были пилотируемыми и вообще были потому, что:
мы тогда гораздо меньше знали о планетах Солнечной системы; например, в то время, когда Гагарин крутился вокруг Земли, а Армстронг делал шаг на Луну, о Венере было известно лишь то, что у неё есть атмосфера и достаточно густая; многие учёные тогда предполагали, что она может оказаться пригодной для жизни;
электроника того времени была все ещё достаточно примитивной, тяжёлой и ненадёжной.
Если бы Венера оказалась пригодной для выхода на её поверхность без скафандра, ну хотя бы на полюсах - первые колонии на ней появились бы ещё в восьмидесятых. Но, увы, это не так. И до тех пор, пока что-то из этих трёх пунктов не изменится, на космос будут выделяться средства по остаточному принципу, как и на всю не обещающую немедленной выгоды науку. А в условиях нехватки средств учёным, в плане получения новых сведений выгоднее запустить несколько сотен АМС, чем одну пилотируемую экспедицию. Такие дела.
Аноним 18/10/14 Суб 06:03:37 #150 №129772 
>>129769
Основная причина в том, что АМС можно отправить в многолетнюю миссию без возврата и с серьёзным риском не долететь, а вот для человека такая хуйня не проканает.
Аноним 18/10/14 Суб 07:57:22 #151 №129774 
>>129772
Ты только последнее предложение прочитал?
Аноним 18/10/14 Суб 15:47:50 #152 №129796 
>>129769
>Какой-либо колонизации Солнечной системы мы не видим и не увидим потому, что:
>1. в этой системе нам известен ровно один уголок ...

Фигня.

Человеки в своем естественном виде жить могут только в африке. А точнее в своем естественном виде они вообще вымерли - выжили только те которые в неестественном.

На всей остальной части земли человеки могут только при помощи технических средств. Даже банальная охота это луки, стрелы , копья и так далее. Чуть северней - еще и огонь. То есть технологии. Примитивные - но человеки к ним шли тысячелетия, между прочим.

Соответственно с этой точки зрения космос отличается от территории нашей раши только необходимым объемом технологий. Что бы жить в раше нужен огонь луки-стрелы, копья, дома деревянные или там чумы, топоры(каменные). Чуть побольше населения - уже сельхоз. для космоса нужны роботы там и всякое такое.

При чем что самое интересное с помощью таких же "космических" технологий люди вынуждены будут жить и на земле. Так как если будут как счас засирать землю и жечь природные ресурсы - тупо все сдохнут.

То есть полюбому что в космосе что на земле нужен будет полный рециклинг. А ископаемые тратить только если чтото новое надо построить, увеличить количество техстредств.
Аноним 18/10/14 Суб 15:52:28 #153 №129797 
>>129796

То есть

а) для большей части территории земли нужно уметь создавать среду обитание в минимально объеме.

б) сейчас человеки создают свою среду обитания в обьеме гораздо большем чем минимальный (города)

в) для жизни на других планетах и на земле в будущем нужно будет уметь создавать среду обитания в объеме еще большем из еще более сырых ресурсов.

То есть выбора то нет - колонизация космоса обязательно случится. Вопрос когда и какие технологии какого уровня для этого нужны.
Аноним 18/10/14 Суб 16:33:19 #154 №129799 
>>129797
>человеки создают свою среду обитания в обьеме гораздо большем чем минимальный (города)
Сейчас среда обитания человечества разрослась до размеров всей планеты. Если раньше каждая деревня обеспечивала себя всем необходимым, то сейчас продукцией одного-двух предприятий пользуется весь мир. И только это позволило снизить цену на хайтек технологии до приемлемого уровня.
В космосе все иначе. Каждая колония должна быть полностью автономной и самовоспроизводимой исключительно своими силами, при этом сохраняя уровень качества и цены как при пользовании глобальной экономикой и планетарной дифференциацией труда.
Аноним 18/10/14 Суб 21:10:13 #155 №129818 
>>129799
> разрослась до размеров всей планеты.

Даже сейчас эмпирический в экономике - экспорт-импорт для страны составляет только порядка 10% ВВП, не более. То есть по прежнему большая часть всего производится локально. То же самое на уровне штата в тех же сша.

Если же в целом, то грубо говоря раньше коровник был целиком на локальном, а сейчас к нему везут всякие там добавки, присадки, катализаторы и так далее. Нечто очень дорогое но небольшое по массе.

> В космосе все иначе. Каждая колония должна быть полностью автономной и самовоспроизводимой исключительно своими силами,

Вообще говоря нет, точнее не совсем. Ну если говорить о ближнем прицеле и солнечной системе.

Каждая колония, точнее группа колоний должна производить на месте 99% всего необходимого по массе. Так как нет проблемы отправлять на марс микросхемы например. Или возить микросхемы с марса. Потому что килограмм микросхем это песец как много и дорого.

Следующий момент - эволюция современного железа(любого). Постепенно весь сложный инждинринг стараются вынести в софт. Хороший пример тут SDR (software defined radio). Когда любое радиоустройство это по сути всего лишь софт, запускаемый на универсальном SDR устройстве (FPGA+процессор общего назначения). Собственно мы это видим в современных мобилах и прочих таких девайсах - все строится вокруг одного SoC.

То есть глобализация развивается в том направлении что стоимость хайтеха уходит в софт(в те же прошивки) и туда же уходят производственные цепочки. А софт перекачать на марс(в колонии) не проблема.

Ну и последнее. 3д печать (технологии аддитивного производства) появились буквально вот-вот и стали возможны только благодаря развитию компутеров. Тут просто переспективы такого скачка технологий для космонавтики даже трудно оценить.

Собственно так что картина вырисовывается такая. Каждая колония будет иметь 3д принтеры для производства почти всего чего угодно. Микросхемы и другие особенно ключевые компоненты, небольшой массы, будут завозить мелкими порциями с земли, а потом и с соседних более развитых колоний. А всякая байда вроде ракетных движков, баков, оборудования и тп будет печататься на месте.

Соответственно дизайны устройств изменятся что бы это отразить. Любой девайс будет состоять нескольких частей. Одни будут более оптимизированы по цене под удобство 3д печати, а другие оптимизированы по массе. Те части что оптимизирванны под печать будут проектироваться из расчета изготовления из доступных на месте материалов.

> сохраняя уровень качества и цены

Понятно что стоимость жизни человека в колониях будет больше чем на земле. Но эта стоимость и на земле везде разная. Вон в сомали все очень дешево например. А в европе рчень дорого. Но стремятся почему то из сомали в европу а не наоборот.

Так и тут, подозреваю что космос будет заселять верхний средний класс и высокооплачиваемые техногики. Даже такая отдельная культура сформируется, когда люди будут гордится тем что тратят на жизнь больше но живут более автономно.
Аноним 18/10/14 Суб 22:49:00 #156 №129829 
>>129768
Нет потому что вечность двигателя определяется не сроком его работы а тем что он может работать сам по себе не черпая энергию из внешних источников. Гравитационное поле луны как раз таким источником и является.
Аноним 18/10/14 Суб 23:14:57 #157 №129831 
>>129769
>в этой системе нам известен ровно один уголок, где человек может прожить без скафандра дольше пяти минут - это планета Земля;
Однако есть как минимум два небесных тела где вполне можно существовать в относительно простых модульных станциях по типу полярных – Луна и Марс.
>2. никто пока не предложил экономически обоснованную причину разворачивать какую либо промышленность или добычу где-либо в космосе. С существующими средствами доставки, даже если астрономы завтра найдут астероид из чистого палладия, добыча и доставка этого палладия на Землю обойдётся дороже, чем стоит местный палладий. Особенно, учитывая, что цена на этот металл сразу же упадёт;
Это вопрос связи и потребностей. 200 лет назад добывать керосиновую жижу со дна моря никому не было нужно, однако появление и распространение ДВЗ резко увеличило спрос на нефтепродукты и сейчас жижу пытаются гнать даже из сланца. Так что дороговизна добычи на начальном этапе может быть скомпенсировано эффектом объёма и развитием ресурсоёмких технологий которые тормозились нехваткой ресурса на земле. Разработка экономической стратегии эксплуатации космоса тормозится тем что
>3. договор об освоении космоса похоронил все политические причины для государств разворачивать какие-либо военные базы или поселения с целью "застолбить" потенциально полезную собственность.
Эта конвенция плохая, негодная. Её надо серьёзно править т.к. на данный момент она представляет собой огромный "кирпич" на въезде в космос.
Так что сейчас препятствий для полноценного космического рывка не так много:
Дурная конвенция, которая ограничивает интерес к космосу кучкой энтузиастов и яйцеголовых.
Отсутствие технологий способных послужить катализатором к поиску дефицитных ресурсов.
Дороговизна пусков, что вызвано отсутствием массовости и следовательно тем что сейчас каждый КА или РН по сути единичный продукт с ручной сборкой и зверскими требованиями по надёжности.
Аноним 18/10/14 Суб 23:20:56 #158 №129834 
> Каждая колония должна быть полностью автономной и самовоспроизводимой исключительно своими силами,
Ноуп. Как раз для "ближних" колоний не проблема щамутить регулярные поставки "провизии" параллельно со сбором их продукции см полярных и шельфовых буровиков-вахтовиков и города крайнего севера
Аноним 19/10/14 Вск 00:25:53 #159 №129841 
>>129831
>ДВЗ
Это че такое? Двигатель внутреннего згорания?
Аноним 19/10/14 Вск 19:03:58 #160 №129889 
>>129834
>Ноуп. Как раз для "ближних" колоний не проблема щамутить регулярные поставки "провизии" параллельно со сбором их продукции

Неа, для космоса это большая проблема же. Закон циолковского, для линейного увеличения delta-v масса растет экспоненциально. А везти надо гораздо больше, потому что воздуха и воды нет.

То есть возникает ситуация когда возможность не возить космонавтам ту же еду устраняет необходимость в вполне значительном числе пусков тяжелых носителей.

В результате если бы была возможность посылать на луну только несколько килограммов в год запчастей на мини-колонию (а топливо, кислород, еда, конструкционные материалы для базы были бы местные) и техника бы не ломалась как сейчас, то лунные базы пилили бы уже давно все кому не лень. Без всяких ядреных двигателей и анобтаниума.
Аноним 19/10/14 Вск 22:32:45 #161 №129911 
>>129889
>А везти надо гораздо больше, потому что воздуха и воды нет.
Как там в 1940х, сантиметровые радары уже изобрели? Проблему регенерации кислорода и воды уже успешно решили лет 60 назад и обкатали в подлодках и орбитальных станциях. Так что проблемой для колонистов в обозримом будущем будет еда до появления развитой гидропоники. И оборудование.
Аноним 19/10/14 Вск 23:51:57 #162 №129921 
>>129911
Таки да. Добывать можно всё что угодно а вот выращивать огурцы - куда более сложная задача. Огромные площади теплиц + надо ебстись с удобрениями..
Аноним 20/10/14 Пнд 21:12:50 #163 №129981 
14138251706380.jpg
Народ, такой вопрос. Насколько близко аппарат может пролететь около нашего светила - солнца? Сохранив при этом работоспособность, частично или полностью.

Какой аппарат был к солнцу наиболее близко?
Аноним 20/10/14 Пнд 21:47:44 #164 №129983 
>>129981
Главная проблема в полетах к Солнцу - это не высокая плотность радиации, а чудовищный расход дельты. Для полёта к Меркурию нужно больше дельты чем для полёта к внешним планетам.
Аноним 20/10/14 Пнд 21:57:36 #165 №129987 
14138278563420.jpg
>>129981
>Какой аппарат был к солнцу наиболее близко?
Helios 2. На 2020-е годы планируется Solar Probe Plus, он предположительно пролетит сквозь внешнюю корону.

Ещё меркурианские аппараты были довольно близко - MESSENGER, Mariner 10, будущий BepiColombo. Но они не предназначены для исследования Солнца.

>>129983
Это всё делается гравитационными маневрами вокруг планет. Улисс, например, аж почти на полярную орбиту таким макаром вывели - единственный зонд, сильно вышедший из эклиптики.
Аноним 21/10/14 Втр 01:37:07 #166 №130006 
Звёздные врата возможны?
Аноним 21/10/14 Втр 01:50:44 #167 №130008 
>>130006
>Звёздные врата возможны?
Сорт оф управляемые червоточины.
Не противоречит ОТО, по крайней мере.
Аноним 21/10/14 Втр 01:51:59 #168 №130009 
>>130008
А есть шанс что власти скрывают прямо сейчас?
Аноним 21/10/14 Втр 01:55:54 #169 №130011 
>>130009
Нет технологий, десу. Нет нормальной теоретической базы. Нужно много энергии, а жалкие лысые обезьяны не могут даже в нормальный термоядерный синтез. Существование ворот, построенных другой цивилизацией, контролируемые властями в тайне - это в /zog/.
Аноним 21/10/14 Втр 01:57:38 #170 №130012 
>>130011
Ну а если у них есть наквадах реактор все устройства типа напиздили?
Аноним 21/10/14 Втр 02:06:01 #171 №130013 
>>130012
>типа напиздили?
У кого, лол? У хренопланетян что ли?
Аноним 21/10/14 Втр 02:08:41 #172 №130014 
>>130013
Ну в ёгипте. Там помимо штуки должны быть и другие штуки.
Аноним 21/10/14 Втр 02:15:12 #173 №130015 
14138433124310.jpg
>>130014
>Ну в ёгипте

Сцк, пздц. Ты не мог еще толще?
Аноним 21/10/14 Втр 16:15:52 #174 №130054 
Слушайте, а если тело с хоть какой-нибудь массой разогнать до световой скорости, это получается это тело будет представлять из себя что-то вроде черной дыры, которая движится со скоростью света? Ведь при световой скорости тело будет иметь бесконечную массу
Аноним 21/10/14 Втр 18:36:42 #175 №130057 
>>130054
>а если тело с хоть какой-нибудь массой разогнать до световой скорости
Не получится.
Аноним 21/10/14 Втр 20:02:46 #176 №130061 
Верно ли, что посторойка пусковой петли - это больше геополитическая проблема, чем инженерная? Банально негде строить: в океане дорого, в африке и азии - нигры и чурки разъебут, вдали от экватора - неэффективно.
Аноним 21/10/14 Втр 21:29:14 #177 №130079 
>>130061
>Верно ли, что посторойка пусковой петли - это больше геополитическая проблема, чем инженерная?
Нет, в Бразилии, например, никаких особо упоротых бабахов нет.
Аноним 21/10/14 Втр 22:43:49 #178 №130088 
>>130054
Да не растёт масса со скоростью. Весишь ты, допустим, 75 кг и разгоняешься до околосветовой скорости. Но при этом поперечная твоя скорость равна нулю и получается, что вдоль направления движения ты весишь 100500 тонн, а поперёк направления движения- 75 кг. К бесконечности в данном случае стремится не масса, а импульс.
Аноним 22/10/14 Срд 13:59:56 #179 №130106 
>>130061
В восточной азии норм же. Китайцы скорей всего и будут пилить, если кто и будет.
Индусы тоже могут в космос, но вряд ли так круто.

>>130079
бразильская_резня_бензопилой.гиф
Аноним 22/10/14 Срд 15:25:14 #180 №130113 
>>129911
>Как там в 1940х, сантиметровые радары уже изобрели? .

Все таки вы школьники меньше фонтастики читайте, и книжек для детей про чудесы технического прогресса. Меньше ушиб мозга будет.

> Проблему регенерации кислорода и воды уже успешно решили лет 60 назад и обкатали в подлодках и орбитальных станциях.

Хоть бы выкипедию почитал прежде чем позорится, а?
Какая "отработана регенерация", ты о чем вообще?

Фильтрами убирают углекислый газ, и всякие человеческие выделения и это все что кое как "отработано". Воду "регенерируют" только в американском сегменте, и пьют ли они ее на самом деле - хз, из остального генерируют кислород. Система генерации кислорода постоянно ломаются, так что приходится переходить на привозной. И это не считая того что все эти фильтры и реагенты тоже надо возить. Это не отработано, это полная противоположность отработано.

Про подлодки это вообще кулстори та еще. У кораблей нет проблем с пресной водой еще со времен отмены парусного флота. А тоннаж атомной подлодки 10-40килотоннон, туда поглотителей углекислоты можно на полвека загрузить, не считая того что реактор, воды целый океан. А самое смешное то что подлодки в кругосветки под водой только в детских советских книжках ходят. А в реальности не только постоянно всплывают полюбоваться пейзажами, но и проводят большую часть плавания над водой. Погружаясь только для тренировок разных. А если и плывут заметно долго под водой, то на перископной глубине. Для который немцы использовали шнорхель что бы воздух сосать еще во 2ю мировую(а как бы еще и не в первую).

> Так что проблемой для колонистов в обозримом будущем будет еда до появления развитой гидропоники. И оборудование.

Массу изначального кислорода и воды, которые ты регенерировать собрался, ты как завозить будешь? Телепортом? А горючее для транспортников базы добывать? Ты пойми, втык флага никому давно не нужен, а значит и базу из которой тыбудешь в скафандре выходить на пару километров влево-вправо никто строить не будет.

А выращивать еду да, проблема N2 - только вот пока даже с кислородом и водой ничего нормально не решили.
Аноним 22/10/14 Срд 16:53:46 #181 №130121 
Тупой вопрос: может ли теоретически безатмосферная планета вращаться настолько быстро, что космонавт при прыжке выйдет на орбиту (любой диаметр, любая масса, главное чтоб гравитация больше 0,1g была)
Аноним 22/10/14 Срд 16:58:46 #182 №130122 
>>130121
Для того, чтобы выйти на орбиту мало одного импульса. С одним импульсом ты вернёшься обратно на поверхность, т.к. траектория замкнутая.
А вообще - только в теории. В реале материю с планеты раскидает раньше, чем она так раскрутится.
Аноним 22/10/14 Срд 17:28:30 #183 №130123 
>>130121
Если она будет вращаться так быстро, что на экваторе гравитация будет уравновешена, то её распидорасит, ибо планета-то находится в гравитационном равновесии. Так что планета просто не сформируется в таком виде. А ещё при приближении к этому пределу скорости вращения планета будет сплющиваться в диск, не знаю как это повлияет.

>>130122
Ну на орбиту родительского тела - запросто, по крайней мере.
Аноним 22/10/14 Срд 17:43:48 #184 №130125 
>>130123
>планета будет сплющиваться в диск, не знаю как это повлияет.
Замедлится вращение, очевидно.
Аноним 22/10/14 Срд 17:45:56 #185 №130127 
>>130125
Это понятно, я имел в виду как оно повлияет на предел скорости вращения, при котором будет вообще что-либо формироваться.
Аноним 22/10/14 Срд 17:48:11 #186 №130128 
>>130127
Тут, боюсь, ОСНЕ много скурпулёзного матана с подсчетом плотностей, масс, распределения материалов и прочего.
Полагаю, достаточно условиться, что даже на треть орбитальной скорости планета вряд ли раскрутится, оставаясь целой.
Аноним 22/10/14 Срд 19:43:30 #187 №130153 
14139926109160.jpg
Переодически заглядываю на стрим с мкс, сейчас увидел неопознанный объект, который был изначально выше, а затем быстро ушел за горизонт. Думал спутник, но сколько наблюдал за стримом, подобного не видел. Такие дела.
Аноним 22/10/14 Срд 19:43:56 #188 №130154 
>>130153
http://space.utema.ru/earth-online/
Каеспе Аноним 23/10/14 Чтв 10:48:25 #189 №130270 
14140469052290.jpg
Никто не видел подобную пикчу, но только в масштабе и штаб по пропорциям все? А лучше функцию какой-нибудь такой кривой, а то летать на уровне "ну кароч там на 10 км наклонять начинаеш хех" и макака сможет. Я еще пытался взять функцию эллипса и посчитать, все одно, хуета вышла.
Аноним 23/10/14 Чтв 11:42:48 #190 №130305 
14140501688880.jpg
>>130153
Может быть, мусор какой-то
Аноним 23/10/14 Чтв 11:44:02 #191 №130306 
>>130270
У Скотта Мэнли был выпуск про это. Пошукай у него на канале
Аноним 23/10/14 Чтв 12:14:56 #192 №130320 
>>130106
> В восточной азии норм же. Китайцы скорей всего и будут пилить, если кто и будет.
Но ведь у китайцев нет места, всё занято людьми, рисовыми полями и гималаями. А если бы могли, никто им помогать не будет, иначе китай будет доминировать в космосе. А китайцы ведь не могут в свои технологии, только пиздить у белых людей.
>>130079
> Нет, в Бразилии, например, никаких особо упоротых бабахов нет.
Тогда им придётся вырубить последние гектары джунглей, грустнота.
Аноним 23/10/14 Чтв 16:56:43 #193 №130409 
>>127536
http://www.glaz.tv/online-webcams/international-space-station-live
Как просматривать эти кино без говноперевода?
Эти англишные переводчики ни немецкую, ни русскую речь не дают нормально послушать.
Есть вообще такие ресурсы с онглайном без перевода?
Аноним 23/10/14 Чтв 19:11:06 #194 №130421 
>>130409
NASA TV, например.
Аноним 23/10/14 Чтв 19:33:06 #195 №130423 
>>130421
Ссыль штоле, а то у меня только Юстримовский эфир находится.
Аноним 23/10/14 Чтв 23:44:09 #196 №130537 
>>130423
Так бы и сказал, что ты имбецил.
http://www.nasa.gov/multimedia/nasatv/index.html
Аноним 24/10/14 Птн 01:41:01 #197 №130548 
14141004610810.jpg
>>130537
Есть ещё? Вообще всё давай, я никак не могу нихрена нагуглить, хз почему, ленивое хуйло не там ищу, наверное.
Аноним 24/10/14 Птн 02:26:32 #198 №130556 
>>130548
Скажи толком, что тебя интересует конкретно?
http://www.youtube.com/user/ScienceAtNASA
http://www.youtube.com/channel/UCAY-SMFNfynqz1bdoaV8BeQ
http://www.youtube.com/user/ESOobservatory
http://www.youtube.com/user/ESA
Аноним 24/10/14 Птн 02:35:32 #199 №130558 
>>130556
Я ОП этого >>126988 треда, бро, но сам не знаю как такие вещи находить кроме тупого просмотра гугола с запросами [онлайн] {космос} {небесное тело}/{название экспедиции}
Аноним 24/10/14 Птн 02:43:50 #200 №130559 
>>130558
http://stereo-ssc.nascom.nasa.gov/beacon/beacon_secchi.shtml

http://sdo.gsfc.nasa.gov/data/

http://www.lmsal.com/solarsites/
Аноним 24/10/14 Птн 02:52:39 #201 №130562 
>>130558
>кроме тупого просмотра гугола с запросами [онлайн] {космос} {небесное тело}/{название экспедиции}
Попробуй больше конкретики и задавайте свои запросы на проклятом буржуйском. Например,
nasa heliophysics, или european southern observatory, или where is the voyagers today например, тогда будете получать более конкретные ответы, вроде этого: http://voyager.jpl.nasa.gov/where/

Пиздец, че вы за битарды, если в гугл толком не можете, идите тренируйтесь
Аноним 24/10/14 Птн 03:21:46 #202 №130565 
>>130562
>where are the voyagers today
slfx
Аноним 24/10/14 Птн 15:55:56 #203 №130620 
Спэйсаны, а рыбы могут в невесомость? Или их плющит конкретно?
Аноним 24/10/14 Птн 16:54:56 #204 №130621 
14141552968250.png
>>130620
Норм все с рыбами. Наши еще на заре космонавтики гуппи запускали. А не так давно японцы делали эксперимент в своем Кибо.
Аноним 24/10/14 Птн 17:14:37 #205 №130624 
Есть ли способы избавляться от лишнего тепла в космосе помимо твердых и капельных радиаторов? Крайние степени наркомании приветствуются.
Аноним 24/10/14 Птн 22:36:31 #206 №130669 
>>130624
Греть заранее охлажденный хладагент, и выбрасывать перегретое вещество за борт. Всё, больше никаких.
Аноним 25/10/14 Суб 02:27:00 #207 №130689 
14141896200440.jpg
>>130624
Зохватывать ледяные астероиды и растапливать их. Нырять в подлёдные океаны на спутниках.
Аноним 25/10/14 Суб 02:34:10 #208 №130690 
14141900505760.jpg
>>130624
А тебе зачем?
Аноним 26/10/14 Вск 12:58:40 #209 №130848 
14143139202240.jpg
Почему шаттл не начинает вращаться как ебанько после отстрела твердотопливных ускорителей? Изменяемый вектор тяги двигателей? Он же тогда немного "вниз" (относительно себя) лететь будет, поэтому он вверх ногами взлетает?
Аноним 26/10/14 Вск 14:09:28 #210 №130853 
14143217689820.jpg
14143217689831.jpg
14143217689862.jpg
>>130848
Аноним 26/10/14 Вск 14:16:01 #211 №130854 
14143221618300.png
>>130848
Движки под углом стоят.
А когда бак полон - его ещё бустеры тащат.
Аноним 26/10/14 Вск 14:58:17 #212 №130857 
>>130624
Перестать вырабатывать лишнее тепло, например.
Аноним 26/10/14 Вск 15:23:42 #213 №130859 
14143262221170.jpg
>>130857
Начать вырабатывать холод и излучать темноту?
Аноним 26/10/14 Вск 15:38:01 #214 №130867 
>>127536
Я так и не понял, как должен работать пузырь альбукерке. Каким образом современные технологии позволяют влиять на пространство-время? Это же не электромагнит повесить и не фотонами бомбить?
Аноним 26/10/14 Вск 15:41:46 #215 №130871 
>>130859
Кстати, да. Омские теории же приветствуются.
Вот от меня:
брать с собой на корабль запас, скажем, рабочего тела с отрицательной температурой.
Его все равно за борт выкидывать, так хоть двойной профит будет.
Аноним 26/10/14 Вск 18:13:00 #216 №130898 
>>130624
Так ли это нужно? Может теория о том что в вакууме теплу некуда будет деваться стльно преувеличена?
К примеру, станция салют 7 после потери управления с Земли замерзла. Космонавты которые туда прибыли для ремонта обнаружили что температура на борту ниже нуля, вся вода замерзла. Никакой разгерметизации и утечки воздуха не было, станция остыла сама по себе, в вакууме.
Или вот ещё пример, Вояджер оснащенные в качестве источника энергии радиоизотопными термоэлектрическими генераторами, которые вырабатывают тепло непрерывно, за много лет так и не расплавились от выделяемого тепла, более того биметалическая пара которая собственно вырабатывает электричество может работать только ели есть разница в температуре.
Аноним 26/10/14 Вск 18:26:42 #217 №130901 
14143372029720.gif
>>130898
>Может теория о том что в вакууме теплу некуда будет деваться стльно преувеличена?
Штобля. Какая ещё теория. Что ты делаишь, прекрати. Тут элементарная школьная физика.

Вопрос не в том, что ему ВООБЩЕ некуда деваться. Вопрос в том, что в вакууме без потери рабочего тела возможен лишь излучательный теплообмен. А это слишком малоэффективно для чего-нибудь, кроме тухлых ритэгов на сраные сотни ватт/киловатты.

Ну можно ещё теоретически теплоаккумуляторы захуярить - по пути на Марс охлаждаем бочку с водой, потом реактор включается для тормозного импульса и нагревает ее обратно. Потом опять долго остывает. Бочку придется нести с собой, конечно. Ну и всё, варианты кончились, остальное - либо радиатор, либо выбрасываем что-нибудь горячее.
Аноним 26/10/14 Вск 21:20:06 #218 №130915 
14143476068770.jpg
Спейсаны, два параллельно выпущенных луча света летят со скоростью света соответственно. Они неподвижны относительно друг друга?
Аноним 26/10/14 Вск 23:08:24 #219 №130917 
>>130915
Каких таких луча летят? Что за объект такой — луч и как это он летит?
Аноним 26/10/14 Вск 23:13:53 #220 №130920 
14143544337550.png
>>130917
Конечно же луч солнышка в космосе. Свет жи не мгновенно распространяется.
Аноним 26/10/14 Вск 23:34:35 #221 №130921 
>>130920
Какой-то у тебя неправильный пекафейс, ну да ладно. Лучи, или кто там у тебя в этой ситуации друг друга не видят.
Аноним 27/10/14 Пнд 18:24:08 #222 №130963 
14144234485960.jpg
Котаны, какие есть новые док. фильмы. С графёном?
Аноним 27/10/14 Пнд 19:33:43 #223 №130971 
>>130963
Соус пикчи, если это из док. фильма с переводом.
Аноним 27/10/14 Пнд 19:49:54 #224 №130973 
14144285948640.jpg
>>130971
Аноним 30/10/14 Чтв 02:24:59 #225 №131572 
А вот зояснити за СЕПАРАТИЗМ ЭЛЕМЕНТОВ.

Ведь когда супернова ебашит, элементы в ней рождаются весьма случайно и получившееся газопылевое облако довольно довольно однородно в своем составе, так ведь?
Так как же потом это гомогенное месиво разделается на фракции? Засчет чего? Ну в сымсле, атомы в одном золотом самородке могут вообще быть из разных звезд, почему же они разом оказались в такой ебичской куче? Как это вообще происходит?
Аноним 30/10/14 Чтв 02:54:04 #226 №131577 
14146268447470.jpg
14146268447511.jpg
>>131572
>элементы в ней рождаются весьма случайно и получившееся газопылевое облако довольно довольно однородно в своем составе, так ведь?
Нет, не совсем так. Вообще процесс уёбывания суперновых довольно загадочная хуйня, пока гадаем на кофейной гуще и пилим моделирования на компах. Все это весьма условные модели и до конца никто этот процесс не вдупляет. Элементы в звезде расположенны слоями тяжелые ближе к центру - легкие на поверхности, при коллапсе там творится ад и израиль, алсо взрывается оно не строго во все стороны поровну.
>Так как же потом это гомогенное месиво разделается на фракции? Оно не разделяется в космосе, скорее всего, а липнет в кучу.
Частично, особотяжелое, слипается в планетезимали.
>Ну в сымсле, атомы в одном золотом самородке могут вообще быть из разных звезд
Самородки, и прочие штуки - это образование уже планетное, в следствии геологии, тектоники и прочих факторов.
Если ты в рэндомных местах накопаешь несколько ведер камней, тебя весьма удивит его разнообразный состав.
Аноним 30/10/14 Чтв 02:56:48 #227 №131578 
14146270080560.jpg
>>131572
>>131577
Вот еще одна веселая картинка от астрофизиков.
Аноним 30/10/14 Чтв 04:40:41 #228 №131585 
>>131577
Окей, охуенные картинки, что-то немного прояснилось, но услоно.
>Самородки, и прочие штуки - это образование уже планетное
Ну, они вроде в в астероидах вских тоже есть, нет?

то есть, если запихнуть кубометр, да даже земной коры, не говоря об астероидах, там можно найти нано-микро-пикограммы разнообразного и трансжелезного атомизированного говна, во всех этих силикатах и глиномесах глинозёмах?
Аноним 30/10/14 Чтв 05:31:38 #229 №131589 
14146362980100.jpg
Есть ли какие-то нехи или явления в космосе, которые противоречат современным теориям и необъяснимы пока что наукой?
Аноним 30/10/14 Чтв 06:04:58 #230 №131590 
>>131589
Собственно, большинство космических объектов именно такое. О них есть только очень грубые представления и модели, которых иногда несколько, и которые иногда меняются на чуть ли не противоположные. Даже "банальные" черные дыры совсем не банальные. Это как с марсианскими каналами, с которыми наебались по полной.

Все потому, что инструментов мало, и они недостаточно точны. Некоторые вещи не могут быть измерены вообще, но таких мало.
Аноним 30/10/14 Чтв 06:06:04 #231 №131591 
>>131590
>Некоторые вещи не могут быть ни измерены напрямую, ни смоделированы по косвенным признакам, но таких мало.
фикс
Аноним 30/10/14 Чтв 08:09:54 #232 №131595 
14146457940400.jpg
Господа учёные, призадумался я на счёт космоса и на счёт того, что люди первым делом решили проверить когда туда полетели.
Вот пламя например на свече будет полусферическое и медузообразное.
Если отжать мокрое полотенце, вода начнёт обволакивать руки, а если её отпустить в свободный полёт, то она пример форму шара.

Но что же будет, если в космосе включить электрошокер?
Вот я нажимаю на клавишу в ожидании увидеть разряд, что же произойдёт?
Вообще ничего не произойдёт, молния будет иметь отличную от привычной земной форму, а может она тоже будет сферической и шокер замкнёт, возможно даже ударив меня током?

Интересны варианты включения шокера как внутри корабля в невесомости, так и в открытом космосе.
Аноним 30/10/14 Чтв 08:18:04 #233 №131596 
>>131595
На уровне "ёбнуть шокером" - вообще нихуя разницы нет, будет то же самое что и на земле. Но в деталях поведение разрядов и плазмы в микрогравитации немного отличается. Вот например, эксперимент проводился с плазмой:
http://knts.tsniimash.ru/ru/site/Experiment_q.aspx?idE=34
Аноним 30/10/14 Чтв 20:18:37 #234 №131693 
14146895176230.jpg
>>131590
Ну а какие явления самые загадочные? Читал недавно про одно недавно обнаруженное, и забыл про что именно. Там ученые говорили, что оно противоречит всем законам физики.
Аноним 30/10/14 Чтв 21:37:17 #235 №131701 
14146942371500.jpg
>>131693
> Читал недавно про одно недавно обнаруженное, и забыл про что именно. Там ученые говорили, что оно противоречит всем законам физики.
По словам астрофизиков, к Земле приближается облако непонятного происхождения, которое стирает все на своем пути. Оно появилось из черной дыры – на расстоянии 28 000 световых лет от земли. Астрономы, наблюдающие за небесным телом, говорят о том, что им удалось обнаружить странный сгусток, который уже окрестили “ сеющим хаос облаком” – он разрушает все на своем пути: кометы, астероиды, планеты и звезды. Теперь он направляется к Земле.

Космический объект, простирающийся на 10 миллионов миль, был обнаружен обсерваторией НАСА Chandra в апреле этого года и, по словам ученых, относится к разряду “кислотного тумана”. Как ожидается, загадочное облако достигнет Земли к 2014 году.

http://2012over.ru/chernaja-dira-unichtozhit-zemlju-v-2014-godu.html
Аноним 30/10/14 Чтв 21:50:06 #236 №131702 
>>131693
Там нечему противоречить, ибо теория в плане экзотических объектов типа TZO или Великого Аттрактора очень умозрительна, там чуть ли не гадание на кофейной гуще идёт.
Аноним 30/10/14 Чтв 23:14:43 #237 №131718 
>>131693
>Читал недавно про одно недавно обнаруженное, и там говорили, что оно противоречит всем законам физики.

С каких пор Рен-Тв выходит в текстовом варианте?
Аноним 31/10/14 Птн 03:58:43 #238 №131761 
Аноны, смотрел я вид с МКС через сайтик и когда станция уходила в тень на экране было полно бликов, но некоторые их них были oche яркие, синие и оранжевые, что это было? Алсо потом эти блики стали полосочками, т.е. верхняя и нижняя их часть постепенно перестала отражаться и остались только серединки.
Я конечно подозреваю, что насмешу вас таким вопросом, но всё же спрошу. Другие то блики не меняли ни цвет не форму.
Аноним 31/10/14 Птн 04:25:32 #239 №131765 
>>131761
Ехал блики через блики.
Как ты предлагаешь разбирать твои галлюцинации, или что там тебе привиделось. Скрин бы хоть снял.
Аноним 31/10/14 Птн 07:23:22 #240 №131770 
Почему запуски водородных ракет дороже керосиновых?
Из-за проблем с хранением водорода?
Аноним 31/10/14 Птн 12:10:48 #241 №131784 
>>131770
Тупо размер ракеты больше.
Аноним 31/10/14 Птн 13:04:37 #242 №131790 
>>131770
Из-за проблем с хранением водорода, да, но и >>131784 тоже важно, габаритные ракеты делать сложно. И вроде бы как двигатели делать сложнее, нагрузка больше, емнип.
Аноним 31/10/14 Птн 18:12:57 #243 №131865 
>>131693
Тёмная материя и энергия, сверхновые, которые по всем расчётам пока не взрываются. Офк чёрные дуры и нейтронные звезды, в которых нейтронов процентов 10, а в центре хуй знает что.
Аноним 31/10/14 Птн 21:13:51 #244 №131923 
Почему на роверах не ставят гусеницы?
Аноним 31/10/14 Птн 21:18:57 #245 №131924 
>>131923
Платина. Потому что если он разуется - то это будет пиздос.
Аноним 31/10/14 Птн 21:34:28 #246 №131925 
>>131923
Тяжело и ненадежно.
Аноним 31/10/14 Птн 21:45:34 #247 №131930 
14147811342050.png
>>131923
Гусеница сбита, движение невозможно
Аноним 01/11/14 Суб 01:32:50 #248 №132081 
>>127786
НЕ ТРОЛЛЬ.
>со сверхсветовой скоростью.
>сверхсветовой
???
>При этом достигнуть размеров СШ ЧД сможет лишь за бесконечное время
>бесконечное время
Т.е. не сможет?
>Для наблюдателя внутри ЧД это будет выглядеть как коллапс, затем мгновенный прилёт античастиц и излучение частиц и полная аннигиляция ЧД.
А,вне чд ,этого события не будет?
>Во время коллапса ЧД гравитация увеличивается и время замедляется
>время замедляется
Но как?
т.о. похуй?
Аноним 01/11/14 Суб 12:48:17 #249 №132176 
>>132081
Удвою вопросы этого адеквата.
Открыл тред чтобы задать ровно те же вопросы и ровно тому же посту.
Аноним 01/11/14 Суб 12:50:45 #250 №132177 
14148354454520.jpg
> 2014
> верить в чёрные дыры
plebs pls
sageАноним 01/11/14 Суб 17:22:44 #251 №132205 
test
>>130000
sageАноним 01/11/14 Суб 20:13:34 #252 №132245 
>>132177
Ээээ...а впрочем.
Аноним 02/11/14 Вск 02:59:12 #253 №132381 
14148863525770.jpg
А у Черных дыр есть орбиты? Они движутся в пространстве, или только вместе с расширением пространства?
Можно ли передать Черной дыре импульс, изменить её орбиту? Есть ли у неё первоначальный импульс от сколапсировавшей звезды?
Аноним 02/11/14 Вск 03:49:08 #254 №132394 
>>132381
Да.
Аноним 02/11/14 Вск 06:59:35 #255 №132404 
14149007751310.jpg
Что будет если выстрелить в невесомости из дробовика?

1.Насколько сильно отбросит стрелка назад? Какая скорость примерно и опасно ли столкнуться спиной с обшивкой,

2.Какой силы будет момент вращения при выстреле? Будет ли это опасно для стрелка, сможет ли он удержать оружие у себя в руках?

3. Будет ли дробь летать дольше и рикошетить от предметов чаще?
Аноним 02/11/14 Вск 07:30:03 #256 №132407 
>>132404
Законы Ньютона - твои друзья. Если бы ты спросил в обратном порядке, я бы решил, что это такой дружелюбный троллинг от олдфага. Лень считать, предположим, у нас сферический дробовик с энергией выстрела, равной силе удара боксёра. Думаю, в реале оно как-то так и обстоит.

1. Читай третий закон Ньютона. Столкнется с обшивкой, конечно, но не сильно, так как сама по себе сила выстрела не настолько мощная, иначе бы стрелков убивало или хотя бы сшибало с ног отдачей, а бронежилет не имел бы никакого смысла.

2. Читай второй закон Ньютона. Ему лучше стрелять от бедра, а ещё лучше - хорошо закрепившись. Вращение тем больше, чем дальше от центра масс будет выстрел. Может закрутить достаточно, чтобы проломить голову или повредить скафандр, если наткнуться на что-то твёрдое и острое. И вообще в этот момент космонавт станет беспомощным и вряд ли удержит оружие, так как ему нужно будет скорей зацепиться за опору какую-нибудь.

3. Читай первый закон Ньютона. От условий Земли отличия только в отсутствии притяжения и, возможно, сопротивления воздуха, если вокруг вакуум. Дробинки, улетающие в космос, будут лететь сколько угодно, рикошет забирает уйму энергии, так что те дробинки, которые не застряли в поверхности, а развалились на мелкую крошку или так и остались в полёте, через 1-2 удара потеряют всю энергию и просто безобидно "поплывут".
Аноним 02/11/14 Вск 13:16:05 #257 №132446 
14149233652880.jpg
Почему на большинстве панорам Марса такой размытый задний план, будто там из-за вечной пылевой бури видимость двести метров? Неужели там постоянно такая пылевая взвесь, что нельзя даже снять с какого-нибудь холма так, чтобы было видно вокруг хотя бы на несколько километров?
inb4 снимали в павильоне
Аноним 02/11/14 Вск 14:22:45 #258 №132467 
>>132404
> .Насколько сильно отбросит стрелка назад? Какая скорость примерно и опасно ли столкнуться спиной с обшивкой,
Закон сохранения импульса у вас ещё не проходили? Выстрел патроном 12 калибра (масса дроби 35 грамм, минимальная начальная скорость, если снаряжать Сунаром - 325 м/с) придаст стрелку (общий вес с ружьём - 83-84 кг) скорость где-то в районе 0,1м/c.
Аноним 02/11/14 Вск 16:43:32 #259 №132549 
>>132446
Ну вообще да. Постоянные ветры, влаги нет, гравитация маленькая – соответственно всегда пыль в воздухе.
Аноним 02/11/14 Вск 18:24:56 #260 №132571 
>>132549
Это что же, выходит, не увидеть нам пейзажей долины Маринера, от которых бы дух захватывало, потому что там уже через пару километров нихрена не видно? ._.
Аноним 02/11/14 Вск 19:32:35 #261 №132586 
Как повлияет на растения меньшая гравитация? увеличатся ли они в размерах?
Аноним 02/11/14 Вск 19:42:14 #262 №132590 
>>132586
Да, повлияет, так же как увеличное содержание ЦэО2 в атмосфере, но все это будет нивелировано малой плотностью атмосферы, так как выяснилось, что без ветровой нагрузки деревья ломаются под собственным весом, ибо вырастают хрупкими.
Аноним 02/11/14 Вск 21:08:34 #263 №132621 
14149517144130.jpg
>>132446
>>132571
Все верно. Но на несколько километров иногда видно, хоть и не очень хорошо.
Аноним 03/11/14 Пнд 03:29:11 #264 №132692 
>>131701
Чет в гiлас
Аноним 03/11/14 Пнд 14:25:32 #265 №132768 
Гамма всплеск это пиздец всему? Можно от него урыться?
Аноним 03/11/14 Пнд 14:30:31 #266 №132769 
>>132768
>урыться?
Можно, только подготовься к земле.
Аноним 03/11/14 Пнд 14:37:59 #267 №132772 
>>132769
А у тебя уже есть доказательства, что мощное гамма излучение проникнет внутрь Солнечной системы и сможет преодолеть магнитное поле планеты?
Аноним 03/11/14 Пнд 14:39:08 #268 №132774 
>>132772
>гамма излучение
>преодолеть магнитное поле
Вернись в обо/сцай, дебилушка.
Аноним 03/11/14 Пнд 14:52:17 #269 №132781 
14150155372380.jpg
>>132772
>гамма излучение блокируется магнитным полем.
Аноним 03/11/14 Пнд 17:33:07 #270 №132820 
14150251874940.jpg
Анан, выглядело бы ночное небо по другому, если бы Земля была расположена в туманности?
Аноним 03/11/14 Пнд 17:51:04 #271 №132823 
>>132768
Самое лучшее и самое единственное средство от гамма всплеска - это расстояние.
Аноним 03/11/14 Пнд 17:55:17 #272 №132824 
>>132820
>выглядело бы ночное небо по другому, если бы Земля была расположена в туманности?

Есть подозрение, что небо было бы темновато. Обезьяны до сих пор бы думали, что их мирок - это и есть вся Вселенная. У Дугласа Адамса было мрачное описание такого мира, аборигены однажды увидели звезды и сошли с ума
Аноним 03/11/14 Пнд 17:56:27 #273 №132825 
Возможно ли выйти на орбиту и сделать хотя бы один виток вокруг, скажем, Фобоса? Или его форма сделает ее слишком нестабильной даже для этого?
Аноним 03/11/14 Пнд 17:59:35 #274 №132826 
>>127786
> неодинаково ведут себя
схуяли
Аноним 03/11/14 Пнд 18:01:56 #275 №132828 
>>132825
> выйти на орбиту Фобоса
у русских, когда они еще не были дураками, такой проект был, например
Аноним 03/11/14 Пнд 18:04:02 #276 №132829 
>>132825
Можно, почему нет. Во Розеттка летает вокруг двойной картохи и ничего.
Аноним 03/11/14 Пнд 18:38:35 #277 №132837 
14150291158190.jpg
Анон, что на внутренней стороне обтекателя? Зачем?
Выручай, все мысли о этом, жизни нет.
Аноним 03/11/14 Пнд 18:53:33 #278 №132839 
>>132837
>Обратите внимание на блоки, расположенные на внутренней стороне обтекателя. Это компоненты акустической защиты (FAP). В то время как обтекатель будет защищать марсоход при старте от аэродинамического давления и нагрева, компоненты FAP оградят космический аппарат от вибраций, создаваемых ракетой при старте.

Хм.
Аноним 03/11/14 Пнд 19:10:49 #279 №132841 
>>132837
Это ПУПЫРЫШКИ же, как на полиэтиленовой упаковке, но побольше.
Аноним 03/11/14 Пнд 20:08:08 #280 №132846 
>>132824
С чего бы? Если она достаточно плотная, чтобы загораживать звезды, то будет тогда рассеивать дохуя света от Солнца и подсвечиваться им.
Аноним 03/11/14 Пнд 20:18:12 #281 №132847 
14150350923860.jpg
>>132828
Но тогда даже концепцию многоступенчатых ракет почти никто никогда не принимал всерьез, е-мое.
Аноним 03/11/14 Пнд 21:17:15 #282 №132852 
Что прочитать из доступного но научного о космосе, космических исследованиях, станциях и так далее?
вообще ничего не читал
Аноним 03/11/14 Пнд 21:26:35 #283 №132854 
>>132852
Начни сначала с википедии.
https://ru.wikipedia.org/wiki/Космическое_пространство
Аноним 03/11/14 Пнд 21:31:01 #284 №132858 
У меня вопрос про Бетельгейзе.
Когда может ёбнуть? Правда ли что на Земле несколько месяцев после взрыва не будет ночи? Будет ли сильная радиация на Земле после этого? И короткий вопрос, какие из ближайших звёзд собираются взрываться?
Заранее спасибо.
Аноним 03/11/14 Пнд 22:03:41 #285 №132866 
>>132858
>Когда может ёбнуть?
завтра, плюс-минус 50 000 лет
>Правда ли что на Земле несколько месяцев после взрыва не будет ночи?
нет.
>Будет ли сильная радиация на Земле после этого?
Да, но только если мы решим устроить сами себе атомный холокост.
Аноним 03/11/14 Пнд 22:37:34 #286 №132870 
>>132846
Вот именно и будет полное ощущение, что вокруг плотная сфера, типа как небесная твердь.
>>132852
Черток "Ракеты и люди", например.
>Когда может ёбнуть?
Может уже, может скоро, может не очень скоро.
> Правда ли что на Земле несколько месяцев после взрыва не будет ночи?
Нет, но светить будет ярко, видно будет даже днем.
>Будет ли сильная радиация на Земле после этого?
Наблюдения показывают, что полюсом Бетельгейзе на нас не направлена, так что никаких серьезных катаклизмов не предвещается.
>плюс-минус 50 000 лет
Плюс - да, минус нет, так как всего 600 световых лет.

Аноним 03/11/14 Пнд 22:37:50 #287 №132872 
>>132858
>>132870
Аноним 04/11/14 Втр 00:48:32 #288 №132884 
>>132590
Ух ты, не знал этого, спасибо.

мимо
Аноним 04/11/14 Втр 01:16:35 #289 №132887 
Помогите вспомнить игру, играл году в 92-94 на какой-то древней пекарне. Имеем космический корабль, который нужно посадить на гору. Графона естественно нет, чёрный экран насколько я помню, корабль и гора визуализированы точечками-чёрточками. Казалось бы всё предельно просто, сверху корабль, снизу гора, но игра была жутко залипательной. Хотеть вспомнить молодость, помогайте
Аноним 04/11/14 Втр 01:32:55 #290 №132890 
>>132866
> плюс-минус 50 000 лет
плюс - писят тыщ, минус - не более 650
Аноним 04/11/14 Втр 02:12:37 #291 №132892 
>>132887
http://moonlander.seb.ly/
Аноним 04/11/14 Втр 16:40:42 #292 №132948 
>>132892
Ух ты, круто!
Аноним 04/11/14 Втр 23:57:31 #293 №132996 
>>127536
почему звезда не превращяется в чёрную дыру до того, как она не сожжёт часть себя? почеум надо ждать пока сколлапсирует ядро? а то странно звучит, когда говорят, что более тяжёлому объекту трудней стать чёрной дырой чем более лёгкому.
Аноним 05/11/14 Срд 00:56:52 #294 №132997 
14151382129640.jpg
>>132996
Потому что звезда - это состояние равновесия, между гравитацией и термоядерными реакциями в ней.

>что более тяжёлому объекту трудней стать чёрной дырой чем более лёгкому.
Нет, так тоже не говорят, ты не правильно истолковал. Ядро звезды становится все тяжелее и плотнее, компактней, по мере выгорания топлива.
Аноним 05/11/14 Срд 00:58:42 #295 №132999 
14151383224580.png
>>132997
Видимо что-то случилось.
Аноним 05/11/14 Срд 10:22:15 #296 №133022 
14151721355460.png
На Земле высота в атмосфере отсчитывается от уровня моря. Это понятно.

Но от чего отсчитывается высота на Венере? Там же нет моря. А на Юпитере?
Аноним 05/11/14 Срд 16:47:42 #297 №133077 
>>132829
Так они параллельным курсом летит, не?
Аноним 05/11/14 Срд 18:08:32 #298 №133085 
>>133022
Ну вообще смотря какие высоты тебя интересуют. Геодезическая высота отсчитывается от условного эллипсоида, который для многих крупных стран свой собственный. Нормальная высота отсчитывается от квазигеоида, который уже не совсем условен, но тем не менее имеет довольно узкое применение. "Высота над уровнем моря" - такой себе термин, довольно примитивный. По факту эта высота отсчитывается от уровенной поверхности потенциала силы тяжести. То есть отвесные линии перпендикулярны этой поверхности в каждой ее точке. Она практически идеально соответствует среднему уровню вод мирового океана. Но ни что не мешает существовать этой поверхности у любых массивных тел, даже тех, что неправильной формы.
Аноним 05/11/14 Срд 18:15:09 #299 №133087 
>>133077
>Так они параллельным курсом летит, не?

Не. ШНЯГА КРУЖИТ ГОЛОВУ КОМЕТЕ И КОНЧАЕТ НА НЕЕ ЗОНДОМ СМОТРЕТЬ БЕСПЛАТНО БЕЗ СМС
http://www.youtube.com/watch?v=Mf1zsACcXc4
Аноним 05/11/14 Срд 20:13:05 #300 №133115 
>>133085
Это возможно, но только в одной плоскости. И дело не в форме, а в том что Фобос очень близко к марсианскому пределу Роша, т.е. с "нижней" стороны Фобоса гравитация Марса почти пересиливает фобосовскую, а с "верхней" нет. В результате траектория становится плохо предсказуемой. Фобос-2 был именно на такой нестабильной орбите, там скорее гало-орбита была, но быстро эволюционировала из-за присутствия Фобоса.
Аноним 05/11/14 Срд 20:13:59 #301 №133116 
>>132829
Таки она своей тягой орбитирует.
Аноним 05/11/14 Срд 20:15:31 #302 №133117 
>>133115
Бля, это тебе >>132825
Аноним 05/11/14 Срд 20:21:55 #303 №133118 
>>133116
Много тягой не наорбитируешь. Тягой она маневры делает.
Аноним 06/11/14 Чтв 02:14:57 #304 №133145 
Аноны, вопрос такой:
если у меня на земле вдруг окажется в распоряжении кусочек солнца со всеми его звездными характеристиками, размером с монету, на сколько километров вокруг все выгорит?
Или просто будет небольшая вспышка слева
Аноним 06/11/14 Чтв 05:16:14 #305 №133149 
>>133145
In the spectral class label, G2 indicates its surface temperature, of approximately 5778 K (5505 °C, 9941 °F)

Т.е. на расстоянии 5-ти метров можно будет руки греть.
Аноним 06/11/14 Чтв 05:42:53 #306 №133151 
>>133149
солнце горит за счёт гравитационного сжатия, т.е. на земле оно разлетица. подозреваю, что небольше чем на пару сотен метров ебанёт.
Аноним 06/11/14 Чтв 05:48:53 #307 №133152 
>>133149
>>133151
Что, даже из центра солнца шарик размера пинга внезапно оказавшийся на земле или у анона над головой не бомбанет на многие километры и пожжет все как атомная бомба?
Разачарован но спасибо за ответы
Аноним 06/11/14 Чтв 07:16:03 #308 №133159 
>>133152
>из центра солнца
Center density 162.2 g/cm3
Center Temperature 1.57×10^7 K
Тебе пиздец.
Что до этих двух дебилов уровня обо/сцай - >>133149 >>133151 - просто без комментариев.
Аноним 06/11/14 Чтв 15:48:51 #309 №133199 
на основании чего они полетели к другим звёздам в фильме "интерстеллар"? фильм не смотрел, но интересно
Аноним 06/11/14 Чтв 18:39:24 #310 №133217 
>>133159
> дебилов уровня обо/сцай
ты в израиле учился? где ты - олигофрне - видишь, что в первом сообщении написано о центре солнца?
Аноним 06/11/14 Чтв 19:58:06 #311 №133231 
>>133217
>ты в израиле учился?
Я кстати учился, рикомендую!
Аноним 06/11/14 Чтв 20:09:04 #312 №133232 
>>133231
Рептилоиды у нас координируются в другом треде.
>>133217
Нахуй ты отвечаешь про поверхность солнца, олигофрен, когда анон спрашивал про ядро?
Аноним 06/11/14 Чтв 20:31:24 #313 №133237 
>>133232
>Нахуй ты отвечаешь про поверхность солнца, олигофрен, когда анон спрашивал про ядро?

>в распоряжении кусочек солнца со всеми его звездными характеристиками

ГДЕ, ИДИОТ? процитируй его пост и выдели часть, где говорица о ядре?
Аноним 06/11/14 Чтв 20:56:26 #314 №133243 
>>133159
Насколько пиздец-то?
То есть будет взрыв? Какой мощности? Гипотетический расчет: вещество из центра солнца (размер пинг-понг) переносится в Москву. Город потерян?
Аноним 06/11/14 Чтв 21:01:13 #315 №133244 
>>133237
Сорри, дурака свалял, надо было точно указать место экстракции вещества на солнце.
Аноним 06/11/14 Чтв 21:11:53 #316 №133248 
>>133243
допустим, что плотность водорода в ядре солнца 200 раз выше чем на земле. значит твой кусочек ОЧЕНЬ БЫСТРО расширится до сферы 250 см диаметром.

помоему это немонго.
Аноним 06/11/14 Чтв 21:21:08 #317 №133252 
>>133248
Допустим, что такая плотность и температура - вполне себе повод ебануть термоядерным взрывом.
Аноним 06/11/14 Чтв 21:24:55 #318 №133253 
>>133248
ой, т.е. 500 см.

если с другой стороны посомтреть, то ядро солднца примерно в 100 раз плотнее тротила(тротил где-то 1.6 г/см3). т.е. если уплотнить тротил в 100 раз, получим 160 грамм на см3. монета имеет объём не больше 1 см3.

т.е., что будет если в москве ебанёт 160 грамм тротила.
Аноним 06/11/14 Чтв 23:00:21 #319 №133265 
>>133253
Ясно короче.
Я то думал будет немалый бабах.
Аноним 07/11/14 Птн 00:33:59 #320 №133286 
14153096396720.png
>>133253
Странный какой-то расчёт.
>ОЧЕНЬ БЫСТРО расширится до сферы 250 см диаметром
>т.е. 500 см
Насколько быстро? А ударная волна от такого расширения? А неебическая температура?
>160 грамм тротила
В тротиловом эквиваленте измеряют энергию взрыва, т. е. к нему переходят от джоулей, а не от массы взрывчатки.
Аноним 07/11/14 Птн 01:09:58 #321 №133288 
>>133286
>В тротиловом эквиваленте измеряют энергию взрыва, т. е. к нему переходят от джоулей, а не от массы взрывчатки.
а ты представь себе что взорвали именно 160 грамм тротила. в нашем случае переводить в энергию просто не надо.

>Насколько быстро? А ударная волна от такого расширения? А неебическая температура?
да какая разница. город оно не снесёт. я сомневаюсь что оно вообще повыбивает все окна на одной улице.
Аноним 07/11/14 Птн 04:56:27 #322 №133297 
>>133252
А чего это в Солнце не ебёт? Тепловыделение ядра порядка ватта на тонну, а у нас 150г всего, которые моментально расширяются и охлаждаются.

Аноним 07/11/14 Птн 06:54:45 #323 №133300 
>>133297
Оно и ебает, постоянно.
Гравитация не дает разлететься всему этому добру.
Аноним 07/11/14 Птн 07:42:47 #324 №133310 
>>133253
>примерно в 100 раз плотнее тротила
Дебил уровня обо/сцай все не уймется.
В следующий раз сразу считай через эфир, неполноценный.
Аноним 07/11/14 Птн 09:32:19 #325 №133317 
>>133297
> Тепловыделение ядра порядка ватта на тонну
> 150г всего, которые моментально расширяются и охлаждаются.
> ебануть термоядерным взрывом
sageАноним 07/11/14 Птн 10:28:57 #326 №133321 
>>133317 - >>133310

Аноним 07/11/14 Птн 14:25:48 #327 №133329 
>>133297
А солнце ебет, просто плотность термоядерных реакций на, скажем, кубометр солнечного ядра достаточно невелика. Другое дело, что 160г вещества с температурой много мульонов кельвинов и давлением в много трильонов паскалей расширится ну очень быстро при этом нагревая окружающее пространство. Условия прямо таки характерные для причин возникновения ударной волны при ядерном взрыве. Килотонны считать надо, теплообмен там, вся хуйня.
Аноним 08/11/14 Суб 01:13:54 #328 №133379 
14153984346760.png
>>133288
>взорвали именно 160 грамм тротила
А почему не гексогена и не чёрного пороха? 0,16 кг тротила дают энергию взрыва 669,44 кДж.
Далее, диаметр мяча для пинг-понга 40 мм, плотность вещества в солнечном ядре порядка 1,5е5 кг/м3. Весить такой шарик должен почти ровно 5 кг.
Дальше считать без бумаги заебло, но я продолжу это неблагодарное занятие.
Аноним 08/11/14 Суб 01:30:24 #329 №133381 
>>133379
Продолжаем воспитательную работу.
Будем считать, что это просто гелий, разогретый до 14е6 К, которые в ядре. Подозреваю, привычная термодинамика при таких адовых условиях будет врать, ну да шут с ней.
Учитывая плотность, получаем, что в нашем шарике содержится 1250 с мелочью моль He. Теплоёмкость у него 20,79 Дж/(K·моль). Перемножая всю эту петрушку, получаем, что в нашем раскалённом шарике содержится порядка 3,65е11 Дж.
В тротиловом эквиваленте это порядка 87 тонн. Не килотонны, но и не 160 г, даже не 5 кг.
И это мы ни слова не сказали об условиях остывания (ударная волна и прочее).
Аноним 08/11/14 Суб 02:31:58 #330 №133387 
>>133381
>воспитательную работу.
Ебать ты метатель бисера перед альтернативно полноценными.
Аноним 08/11/14 Суб 03:25:59 #331 №133390 
>>133387
Интересно жи насколько ебнет и как будет выглядеть
Анон с вопросом
Аноним 08/11/14 Суб 05:19:59 #332 №133394 
>>133390
>Анон с вопросом
Не о тебе речь, ты просто невежда.
Об отметившихся тут дебилах с охуительными расчетами кефирного уровня.
Аноним 08/11/14 Суб 07:25:46 #333 №133396 
>>133381
>1250 с мелочью моль He. Теплоёмкость у него 20,79 Дж/(K·моль).
1250 дж на 1000 моль? 20.791.250 = 25.9875 дж
Аноним 08/11/14 Суб 15:59:22 #334 №133442 
>>133396
>1250 дж на 1000 моль?
Wut? Молярная теплоёмкость у гелия 20,79 Дж/(K·моль), т. е. чтобы нагреть моль гелия на один кельвин надо потратить этот самый 21 без малого джоуль. У нас 1250 моль, нагреты до 14 млн К.
Аноним 08/11/14 Суб 21:40:28 #335 №133494 
>>133394
Тред разнообразных вопросов о Вселенной, жизни и всего такого.

Спрашиваем то, за что в других местах выдают путёвку в биореактор.
>Невежда
Nice
Аноним 09/11/14 Вск 06:10:43 #336 №133514 
>>133381
> И это мы ни слова не сказали об условиях остывания
Остывание может как раз ослабить взрыв, мощность излучения гигантская, большей частью рентген, который преобразуется в тепло где-то далеко от эпицентра. С другой стороны, вещество сильно сжато, расширение усилит ударную волну но я хз, может, будет охлаждать, газ он такой, охлаждается при расширении
Аноним 09/11/14 Вск 11:16:58 #337 №133526 
Объясните мне популярно, почему время для астронавта, летящего с высокой скоростью, замедляется.
Аноним 09/11/14 Вск 14:13:12 #338 №133544 
>>133526
Согласно релятивистским законам теории относительности.
Аноним 09/11/14 Вск 15:42:21 #339 №133550 
Космач, а правильно получается, что температура отражает кинетическую энергию частиц материи? И получается теоретического предела у температуры нет? Ведь если все частицы будут двигаются со скоростью 99,9999999% с, то соответственно их масса также будет гораздо выше, нежели в состоянии покоя. Или я что-то неправильно понимаю?
Аноним 09/11/14 Вск 20:17:00 #340 №133589 
>>133550
Ну есть такая планковская температура 10^96 вроде, дальше теория не работает. Ну и ещё стоит сказать, что температура пропорциональна квадрату скорости, поэтому метеориты пиздецки раскаляются и нагревают всё вокруг при падении.
Аноним 09/11/14 Вск 23:16:11 #341 №133617 
>>127536
Что было до большого взрыва лел ?
Аноним 09/11/14 Вск 23:18:08 #342 №133618 
14155642885150.jpg
>>133617
Ничего.
Аноним 09/11/14 Вск 23:30:32 #343 №133620 
>>133618
А потом пришёл Аллах и сотворил всё сущее?
Аноним 09/11/14 Вск 23:39:51 #344 №133622 
>>133620
http://www.youtube.com/watch?v=Z7k9zaqYAsM
1 В начале сотворил Бог небо и землю.
2 Земля же была безвидна и пуста, и тьма над бездною, и Дух Божий носился над водою.
3 И сказал Бог: да будет свет. И стал свет.
4 И увидел Бог свет, что он хорош, и отделил Бог свет от тьмы.
5 И назвал Бог свет днем, а тьму ночью. И был вечер, и было утро: день один.
6 И сказал Бог: да будет твердь посреди воды, и да отделяет она воду от воды. [И стало так.]
7 И создал Бог твердь, и отделил воду, которая под твердью, от воды, которая над твердью. И стало так.
8 И назвал Бог твердь небом. [И увидел Бог, что это хорошо.] И был вечер, и было утро: день второй.
9 И сказал Бог: да соберется вода, которая под небом, в одно место, и да явится суша. И стало так. [И собралась вода под небом в свои места, и явилась суша.]
10 И назвал Бог сушу землею, а собрание вод назвал морями. И увидел Бог, что это хорошо.
11 И сказал Бог: да произрастит земля зелень, траву, сеющую семя [по роду и по подобию ее, и] дерево плодовитое, приносящее по роду своему плод, в котором семя его на земле. И стало так.
12 И произвела земля зелень, траву, сеющую семя по роду [и по подобию] ее, и дерево [плодовитое], приносящее плод, в котором семя его по роду его [на земле]. И увидел Бог, что это хорошо.
13 И был вечер, и было утро: день третий.
14 И сказал Бог: да будут светила на тверди небесной [для освещения земли и] для отделения дня от ночи, и для знамений, и времен, и дней, и годов;
15 и да будут они светильниками на тверди небесной, чтобы светить на землю. И стало так.
16 И создал Бог два светила великие: светило большее, для управления днем, и светило меньшее, для управления ночью, и звезды;
17 и поставил их Бог на тверди небесной, чтобы светить на землю,
18 и управлять днем и ночью, и отделять свет от тьмы. И увидел Бог, что это хорошо.
19 И был вечер, и было утро: день четвёртый.
20 И сказал Бог: да произведет вода пресмыкающихся, душу живую; и птицы да полетят над землею, по тверди небесной. [И стало так.]
21 И сотворил Бог рыб больших и всякую душу животных пресмыкающихся, которых произвела вода, по роду их, и всякую птицу пернатую по роду ее. И увидел Бог, что это хорошо.
22 И благословил их Бог, говоря: плодитесь и размножайтесь, и наполняйте воды в морях, и птицы да размножаются на земле.
23 И был вечер, и было утро: день пятый.
24 И сказал Бог: да произведет земля душу живую по роду ее, скотов, и гадов, и зверей земных по роду их. И стало так.
25 И создал Бог зверей земных по роду их, и скот по роду его, и всех гадов земных по роду их. И увидел Бог, что это хорошо.
26 И сказал Бог: сотворим человека по образу Нашему [и] по подобию Нашему, и да владычествуют они над рыбами морскими, и над птицами небесными, [и над зверями,] и над скотом, и над всею землею, и над всеми гадами, пресмыкающимися по земле.
27 И сотворил Бог человека по образу Своему, по образу Божию сотворил его; мужчину и женщину сотворил их.
28 И благословил их Бог, и сказал им Бог: плодитесь и размножайтесь, и наполняйте землю, и обладайте ею, и владычествуйте над рыбами морскими [и над зверями,] и над птицами небесными, [и над всяким скотом, и над всею землею,] и над всяким животным, пресмыкающимся по земле.
29 И сказал Бог: вот, Я дал вам всякую траву, сеющую семя, какая есть на всей земле, и всякое дерево, у которого плод древесный, сеющий семя; - вам сие будет в пищу;
30 а всем зверям земным, и всем птицам небесным, и всякому [гаду,] пресмыкающемуся по земле, в котором душа живая, дал Я всю зелень травную в пищу. И стало так.
31 И увидел Бог все, что Он создал, и вот, хорошо весьма. И был вечер, и было утро: день шестой.
(Автор этого поста был предупрежден.)
Аноним 10/11/14 Пнд 00:00:26 #345 №133623 
>>132081 >>132176
Слоуответы...

>сверхсветовой
Пространство расширяется в каждой точке, соответственно там где была один объём через некоторое время станет два, потом четыре, потом восемь. Потому удалённые точки могут удаляться со сверхсветовой скоростью, т.к. собственная скорость их движения не растёт - растёт со скоростью превышающей световую длина пути между точками.

>время замедляется
Во время коллапса гравитационное поле увеличивается и согласно ОТО для стороннего наблюдателя время замедляется, но достигнуть сферы Шварцшильда, когда время в дыре стоит полностью, дыра сможет только за бесконечное время - сжимаясь она приближается к горизонту событий всё медленнее и медленнее, т.к, время всё замедляется и замедляется. Соответственно излучение из дыры будет происходить, но с сильнейшим красным смещением.
Согласно же тории Хокинга об испарении чёрных дыр, виртуальные частицы-античастицы, рождённые рядом с дырой по разному ведут себя и частица вылетает из окрестностей ЧД, а античастица падает на неё. Таким образом, ЧД аннигилируется виртуальными античастицами и испускает волны. При этом при аннигиляции она теряет часть энергии, т.е. часть массы, т.о. радиус сферы Шварцшильда для неё немного уменьшается и время немного ускоряется во время акта поглощения античастицы. Очевидно, что со временем устанавливается равновесие между испускаемой чёрной дырой энергией в виде излучения и массой в виде уже не виртуальных частиц и ускорением времени из-за уменьшения размеров сферы Шварцшильда и замедлением времени в результате гравитационного коллапса. В результате получается не совсем чёрная дыра, которая существует в равновесии, время в которой идёт очень медленно для стороннего наблюдателя, но которая испускает энергию.

Со временем же испарение ЧД происходит всё быстрее, равновесие нарушается и ЧД взрывается, Однако как нет в нашей вселенной остатков красных карликов, потому как смерть красных звёзд наступит лишь через десятки, а некоторых сотни миллиардов лет, так и мы не можем стать свидетелями взрывов ЧД - они находятся на масштабах триллионов лет и больше.

Для внутреннего наблюдателя происходит коллапс, потом накачка извне античастицами и взрыв - всё за считанные мгновения по его часам.

Так я понимаю теорию. Внимательно выслушаю комментарии других диванных космологов.
Аноним 10/11/14 Пнд 00:04:32 #346 №133624 
>>133085
В России уровень моря - это то ли высота относительно нулевой отметки кронштадского хренометра, то ли относительно международного хренометра, вычисленная как высота кронштадского относительно мирового плюс высота относительно кронштадского.
Аноним 10/11/14 Пнд 12:30:40 #347 №133650 
14156118408830.png
Почему Буран не пидорасит в указанном направлении?
Аноним 10/11/14 Пнд 12:42:28 #348 №133651 
>>133650
Немного при старте заваливает, но автоматика компенсирует.
Аноним 10/11/14 Пнд 15:33:21 #349 №133659 
>>133650
Насколько я помню и у основного блока Энергии и у ускорителей есть УВТ.
Аноним 10/11/14 Пнд 16:03:46 #350 №133663 
>>133620
Аллах не нужен.
Объясняю на пальцах:
Если ты вводишь в систему Аллаха, который все сотворил, то возникает вопрос, что было до Аллаха.

Таким образом Аллах не является ответом на вопрос "что было до".
Аноним 12/11/14 Срд 10:21:19 #351 №133927 
14157768794780.jpg
Аноны, я нихуя не понял.
Зачем человечеству нужно было запускать Вояджер в космические ебеня? Это ведь очевидно проигрышный проект. Всем было понятно что он улетит в черные неведомые ебеня и там сгинет безвозвратно. Какие данные он смог передать? То, что солнечная система где-то кончается? Ну охуеть теперь, вот это новость. Какая вообще с этого польза? Мне, тебе, соседу снизу? Да никакой. Не понимаю, как люди, которые принимают важные экономические решения, спустили кучу денег в никуда, совершенно без вменяемой цели. И эти люди что-то мне говорят про экономику?

tl:dr нихуя Вояджер нужен?
Аноним 12/11/14 Срд 10:35:34 #352 №133928 
>>133927
Это зеленоватый вопрос о нужности фундаментальной науки, он для отдельного треда, а лучше для /sci/, где тебя лучше поймут
Аноним 12/11/14 Срд 10:36:30 #353 №133929 
>>133927
Потому что это охуенно
sageАноним 12/11/14 Срд 11:02:24 #354 №133931 
>>133928
>он для отдельного треда,
Ой, не надо, пожалуйста, эти платиновые треды про "космос нинужен" заебали дальше некуда.
Аноним 12/11/14 Срд 11:08:15 #355 №133933 
>>133927
На вот, просвещайся.
Там все ответы на вопросы о пользе вояджеров, выраженных в пользе, килобаксах и прочей шняге.
http://spinoff.nasa.gov/Spinoff2013/pdf/Spinoff2013.pdf
Аноним 12/11/14 Срд 22:31:16 #356 №134673 
Анон а как будет выглядеть мир/космос если смотреть из чёрной дыры?
Аноним 12/11/14 Срд 22:31:46 #357 №134674 
>>134673
Запятую при обращении пропустил, соррьки
Аноним 12/11/14 Срд 22:39:39 #358 №134677 
>>134674
>>134673
Но она не будет выглядеть.
Это Тьма, и некая точка, из которой ебашит лютый, бешеный поток сознания гамма-излучения вперемежку с наблюдателями, окда? разодранными до элементарных частиц.

Но все это продлится не оче долго, потому что скоро эта точка прийдет в состояние Тепловой Смерти.
Аноним 12/11/14 Срд 23:55:30 #359 №134703 
Аноны, по советуйте телескоп более менее годный до 10 к деревянных, желательно в ДС с адресками магазинов.
Аноним 12/11/14 Срд 23:56:33 #360 №134704 
>>134703
самофикс, посоветуйте.
Аноним 13/11/14 Чтв 00:07:55 #361 №134706 
Хокинги, помогите у меня после всех этих поп-научных книжек и фильмов по дискавери образовалась каша по поводу измерений. Я попросту не въезжаю, Земля трехмерная? А за пределами Земли космос - двухмерный, как было сказано в фильме, пространство и время, как это понимать? Космос плоский? Мы просто летим вперед? Или мы можем лететь вниз, вверх или в любом другом направление? Как выстроена Вселенная? Это раздувающийся шар? Поясните пожалуйста.
Аноним 13/11/14 Чтв 00:12:11 #362 №134707 
14158267311010.jpg
>>134703
в разделе этом больше страницы одной
Аноним 13/11/14 Чтв 00:23:45 #363 №134709 
>>134706
> Земля трехмерная? А за пределами Земли космос - двухмерный, как было сказано в фильме
Либо жопой ты слушал, либо истинно каша в голове твоей.

Есть пространственно-временной континуум, с тремя пространственными измерениями и одним - временнЫм. Все что в космосе есть - имеет ту же размерность. людей заглянувших под спойлер сойдут с умахотя некоторые утверждают что существуют объекты имеющие большее количество измерений, но при этом мы из нашего космоса можем наблюдать только их четырехмерные проекции, как максимум
Аноним 13/11/14 Чтв 00:25:45 #364 №134710 
>>134706
а еще загугли световой конус и сломай себе мозг об колено
Аноним 13/11/14 Чтв 00:37:42 #365 №134716 
>>134703
Тебе сюда
https://2ch.hk/spc/res/126271.html
Обрати внимание на шапку.
Аноним 13/11/14 Чтв 00:40:04 #366 №134719 
>>134716
о хорошо
Аноним 13/11/14 Чтв 03:06:24 #367 №134740 
Ребят, объясните, что такое излучение Хоккинга и радиус Шварцшильда, и что такое Релятивистская струя, только попроще, я тупой? И подскажите есть ли интерактивные звездные карты онлайн, с фотографиями и подписями.
Аноним 13/11/14 Чтв 05:17:56 #368 №134751 
>>134740
> излучение Хоккинга
Из черной дыры даже излучение вырваться не может, но благодаря квантовым эффектам таки вырывается
> радиус Шварцшильда
Радиус, до которого надо сжать объект, чтобы тот превратился в черную дыру. Даже у Земли он есть, несколько сантиметров. Даже у твоей мамки.
> Релятивистская струя
Струя с околосветовой скоростью истечения
Аноним 13/11/14 Чтв 06:29:43 #369 №134757 
>>134751
>Радиус, до которого надо сжать объект, чтобы тот превратился в черную дыру. Даже у Земли он есть, несколько сантиметров. Даже у твоей мамки.

То есть, если земля сожмется до нескольких сантиметров, она превратится в ЧД?
Ещё вопрос, солнце в ЧД не сможет, так как массы не хватает? Это что значит? Что нужно Солнцу, чтобы стать ЧД?
Аноним 13/11/14 Чтв 10:55:09 #370 №134791 
>>134757
Тебе же только что, блеать, ответили. Прямо в посте, который ты цитируешь. Ептумать, да в самой цитате содержится ответ на твой вопрос.
Аноним 13/11/14 Чтв 10:55:38 #371 №134792 
>>134740
Привет, ты для начала постарайся прочитать три статьи в педивикии, а потом задай вопросы, если что-то стало не понятно или требует дополнительных разъяснений, подробно отвечать на твои вопросы - значит очень долго топтать клавиши, пожалей анонов.
>>134757
>То есть, если земля сожмется до нескольких сантиметров, она превратится в ЧД
Да
>Это что значит? Что нужно Солнцу, чтобы стать ЧД?
Это значит, что у Солнца недостаточно плотное и недостаточно тяжелое ядро, что бы после выработки топлива оно сколлапсировало в ЧД.
Аноним 13/11/14 Чтв 11:16:53 #372 №134800 
14158666130380.jpg
>>133928>>134791
Уважаемые анонимные любители космических полётов, каждый раз, когда тут кормят говном издеваются или посылают подальше вопрошающего, умирает одна лоли-лисичка. Ну ладно, пусть тон "объясняю клиническому идиоту" просто заставляет её чихнуть но с соплёй!. Не надо забывать о сути треда. Если кто забыл, пусть нажмёт Home на клавиатуре и почитает самый первый пост этой нити.

>>133544
Но тут я всё же проиграл.

мимотрансфернул
Аноним 13/11/14 Чтв 12:05:54 #373 №134824 
>>134800
>>134792
Добра Вам, няши. И последний вопрос, а почему с МКС иногда не видно звезд, из-за другой среды?
Хотя нет, ещё один, а есть здесь астрофизики, ну в смысле не просто любители, а те, у кого космос-профессия?
Аноним 13/11/14 Чтв 12:13:27 #374 №134826 
>>134824
Из-за динамического диапазона камер (нельзя снимать одновременно яркое и темное).
Есть, некоторые тут в аэрокосмосе работают/учатся, либо в смежных отраслях.
Аноним 13/11/14 Чтв 12:18:17 #375 №134827 
14158702971810.jpg
>>134824
>а почему с МКС иногда не видно звезд
Звезды с МКС видны. Просто Земля намного ярче и динамический диапазон камер не позволяет снять и Землю и звезды.

Астрономов, или астрофизиков тут нет. Уж очень редкая специализация.
Есть несколько астрономов-любителей.
И мелькало парочка анонов, которые работают в шарагах, имеющих отношение к созданию космических аппаратов, но не инженеры и их давно не видно.
Аноним 13/11/14 Чтв 14:02:45 #376 №134856 
Парни, а напомните почему у звезд непрерывный спектр, не считая линий поглощения?
Вроде у газа спектры дискретные же, нет?
Сильно не бейте, я так-то оптик, но университет 2 года назад закончил, по специальности уже полтора года не работаю.
Аноним 13/11/14 Чтв 14:12:40 #377 №134860 
>>134856
Потому что кроме излучения разных ионизированных элементов звезда в основном излучает во всем спектре благодаря термоядерному синтезу.
Аноним 13/11/14 Чтв 14:35:20 #378 №134862 
http://esamultimedia.esa.int/images/huygens_alien_winds_descent.mp3

>Several sound samples, taken at different times during the descent, are here combined together

Есть полная версия?
Аноним 13/11/14 Чтв 15:02:09 #379 №134864 
14158801293240.jpg
>>134862
Конечно, все эти данные доступны в Planetary Sscience Archive.
http://atmos.nmsu.edu/PDS/data/hphasi_0001/DATA/PWA/
Дамп данных с пакета датчиков PWA (Permittivity, Wave & Altimetry) инструмента HASI (Huygens Atmosphere Structure Instrument).
Привязка доступна через траекторные данные:
http://atmos.nmsu.edu/PDS/data/hpdtwg_0001/DATA/

Описание того, как с этим работать (форматы, побочные условия и т.п.)- здесь:
http://atmos.pds.nasa.gov/data_and_services/atmospheres_data/Huygens/Huygens.html
Аноним 13/11/14 Чтв 16:28:30 #380 №134876 
>>130624

Тепло преобразовывать в электроэнергию, энергию излучать магнетроном или лазером
Аноним 13/11/14 Чтв 17:40:37 #381 №134882 
>>134876
В твоём манямирке второй закон термодинамики отменили?
Аноним 13/11/14 Чтв 19:14:32 #382 №134924 
>>133526
Ты неправильно задаешь воброс.
Надо не "почему?", а "зачем?"

Зачем у релятивистского космонавта время замедляется, он что, дурак?

Короче, толсто, садись, два.
Аноним 13/11/14 Чтв 19:18:27 #383 №134926 
14158955073630.png
>>134856
Линии излучения уширяются и перекрываются, очевидно же
Аноним 13/11/14 Чтв 19:55:50 #384 №134936 
14158977501220.jpg
14158977501241.jpg
>>127536
Русские доставляют свои ракеты к старту горизонтально, а потом ставят на стартовый стол.
Американцы доставляют к старту свои ракеты вертикально на кроулере. КАК ОНИ ИХ ВЫ ГРУЖАЮТ НА СТАРТОВЫЙ СТОЛ С НЕГО?
Аноним 13/11/14 Чтв 19:57:11 #385 №134937 
>>134936
>ВЫ ГРУЖАЮТ
Зна чение знаю, но это слово почему-то в спам-листе. Какого хуя?
Аноним 13/11/14 Чтв 19:57:44 #386 №134938 
>>134937
>Зна чение
И оно, блядь тоже, лол.
Что за поебень?
Аноним 13/11/14 Чтв 20:01:06 #387 №134939 
>>133650
Посчитано всё. И аэродинамика системы, и то, что при старте основная масса в Энергии, и УВТ, которого хватает после отделения зенитов. И очень удобный автоматический gravity turn, лол.
Аноним 13/11/14 Чтв 20:03:58 #388 №134940 
>>134936
Ганса и Франца сейчас не используют. А когда использовали - у тебя на пике ответ на твой же вопрос: он перевозит ракету прямо с куском стартового стола, и подъезжает под пьедестал, ставя на него всю конструкцию.
http://en.wikipedia.org/wiki/Mobile_Launcher_Platform
Аноним 13/11/14 Чтв 20:05:12 #389 №134941 
>>134937
Пит буль
Аноним 13/11/14 Чтв 20:15:46 #390 №134945 
>>133650
Буран весит 80 тонн, а Энергия на старте около 2400 тонн. Ну и тяги там запас такой, что оно даже при одной сдохшей боковушке на орбиту выходило, зверь-машина. УВТ само собой.
Аноним 14/11/14 Птн 01:36:56 #391 №135000 
>>134945
Соглашусь. Это всё же не поспешно на коленке запиленная Н-1 с тридцатью движками без наземных испытаний, отказ движка которой гарантировал провал запуска. Это SERIOUS BUSINESS.
Наш шаттл был круче, но не смог им быть, т.к. развалился СССР.
Да и хер бы с ним, Спейс Шаттл показал неэффективность многоразового космоплана на этой вехе развития технологий. Но Энергию жалко. Такая-то йоба, способная выводить похлеще Сатурна-5 в определённой модификации (Вулкан). Вложить бы бабла да вывести тороидальную станцию с псевдогравитацией на орбиту. Сразу бы решились проблемы с вымыванием кальция из костей из-за невесомости у космонавтов.
Но нет, всем похуй на космос. Давайте лучше скупать новые гейфончики и лайкать друг дружку в инстраграме.
Тьфу, блядь.
Аноним 14/11/14 Птн 02:23:32 #392 №135011 
14159210129320.png
>>135000
Вы обещали мне закон Мура и аплоад в виртуальную реальность к анимешным лолям-коммунисткам к 2000 году, но вместо этого у нас есть только самоходный перфоратор на Марсе и табличка с именами каких-то поцев на Итокаве.
Аноним 14/11/14 Птн 08:34:03 #393 №135037 
14159432430920.png
Посоны, у меня такой вопрос. Чому люди не ебнут по Венере, чтобы на ней наступила ядерная зима? И какого хуя она до сих пор не наступила, если там и без того всё небо в тучах?
Аноним 14/11/14 Птн 09:43:45 #394 №135049 
>>135037
>Чому люди не ебнут по Венере, чтобы на ней наступила ядерная зима?
Не за чем.
>если там и без того всё небо в тучах?
Парниковый эффект, слышал про такое?
Аноним 14/11/14 Птн 10:42:42 #395 №135058 
>>135037
Тот солнечный свет что все таки туда попадает уже обратно не выбирается. Поэтому там ад, как в пизде твоей мамаши шлюхи.
Аноним 14/11/14 Птн 14:26:25 #396 №135117 
>>135037
But нахуя?
Аноним 14/11/14 Птн 19:27:19 #397 №135209 
Громкий ли шум в солнечном ядре?
Аноним 14/11/14 Птн 20:29:53 #398 №135223 
>>135209
Громкий ли шум в центре термоядерного взрыва?
Аноним 17/11/14 Пнд 12:45:48 #399 №135854 
14162175487270.jpg
Спейсаны, а на поверхности Луны ионные движки будут работать? Вакуум же. Запилить взлетно-посадочный модуль на холлах + РИТЭГ или летающую над повехностью Луны хуитку, подобно катеру на воздушной подушке но с ионниками. Можно будет быстро летать из А в Б не глотая едкую пиль и не портить при этом ландшафт :3
Аноним 17/11/14 Пнд 12:52:07 #400 №135855 
>>135854
>ионные движки
Тяга - говно.
Аноним 17/11/14 Пнд 13:01:46 #401 №135856 
14162185065920.jpg
>>135855
А как же ТЭМ?
Аноним 17/11/14 Пнд 13:07:59 #402 №135857 
>>135856
>Масса 20 тонн
>Суммарная тяга 18 Ньютонов

Ты меня троллируешь что ли? Ионные двигатели только как межорбитальные буксиры, или для полетов в космосе. Для отрыва от поверхности они не годятся от слова совсем.
Аноним 17/11/14 Пнд 13:40:15 #403 №135864 
>>135857
ТЭМ можно за собой на тележке возить.
Аноним 17/11/14 Пнд 13:42:27 #404 №135865 
>>135857
Нет,не тралль. Итого, даже гирлянда этих пуколок сама себя не поднимет над поверхностью Луны? Тогда и ТЭМ будет ползти как черепаха. Хуёво.
Аноним 17/11/14 Пнд 14:09:56 #405 №135868 
>>135865
У ионников отличный удельный импульс, но очень маленькая тяга. Имеет смысл их использовать на дальних миссиях. Dawn, к примеру
Оторвать эту хуйню от земли на собственной тяге совершенно нереально.
Нужно приложить 1 Н, что бы преодолеть силу гравитации на уровне моря, для тела массой в 100 грамм. А тут 20 тонн и 18 ньютонов, все равно, что запускать асфальтоукладчик в космос щелбаном.
Аноним 17/11/14 Пнд 14:17:02 #406 №135870 
>>135865
>Тогда и ТЭМ будет ползти как черепаха. Хуёво.
Разгоняться он будет как черепаха. Зато долго. Очень.
Аноним 17/11/14 Пнд 15:59:30 #407 №135882 
Сап, спейсач, имею вопрос уровня "гуманитарий":
Очень часто слышу, дескать современная наука не имеет двигателей, позволяющих человеку долететь до того же Марса, например. Упустим всяческие доп. составляющие, вроде радиационного поля, долгого полета и проч. сопутствующих проблем. Допустим, это решено.
Почему невозможно долететь, используя пертурбационные маневры?
Заранее спасибо.
Аноним 17/11/14 Пнд 16:09:21 #408 №135885 
>>135882
Имеет, почему же. Обычные ЖРД. Просто с ними придется огромную йобу тащить на орбиту и выделять на это гигантское бабло времен лунной гонки, а профит марсианской экспедиции не настолько очевиден, чтобы так вкладываться.

> Почему невозможно долететь, используя пертурбационные маневры?
Какие-какие маневры? Если ты о гравитационных, то все эти петли вокруг других планет и полеты между точками лагранжа не всегда возможны, не всегда экономят дельту и очень длительны, а у нас человеки на борту. До Марса проще лететь напрямую, он относительно близко.
Аноним 17/11/14 Пнд 18:42:29 #409 №135903 
>>135882
>Упустим всяческие доп. составляющие, вроде радиационного поля, долгого полета и проч. сопутствующих проблем. Допустим, это решено.
>Почему невозможно долететь, используя пертурбационные маневры?

Почему упустим? Это все "слышал звон но не знаю где он". Как уже ответили ЖРД есть. Но проблема (окромя того что это будет огромная, очень дорогая, и соответственно ненадежная иоба) именно в том что не удается пока создать аппарат который будет надежно работать все время миссии. Которое составляет порядка 2.5 лет. И не угробит космонавтов где нибудь например во время стыковки на марсианской орбите. И это не МКС где в следующий грузовик можно срочно сунуть необходимую запчасть - тут такой возможности в принципе нет.

Соответственно есть план используя не-ЖРД движки долететь на марс где то за два месяца на большой скорости, соответственно сократив общее время миссии и избавившись таким образом от проблем с надежностью. Вот собственно это скорее всего и имеют в виду когда говорят что "нет движков". Потому что таких движков действительно еще нет. хотя есть наметки что лет через 5 будет рабочий VASIMIR.
Аноним 17/11/14 Пнд 19:22:53 #410 №135911 
>>135885
>>135903

Спасибо, посоны.
Аноним 17/11/14 Пнд 21:07:59 #411 №135929 
14162476795820.jpg
Раз у Земляшки неравномерная гравитация, то это должно влиять на атмосферное давление и плотность воздуха в соответствующих местах? То есть самолетам например становится проще/сложнее летать в разных частях Земли?
Аноним 18/11/14 Втр 13:13:15 #412 №135980 
Ананасы, поясните за тезис:
Позитрон движется в обратном направлении по стреле времени.
Источник проебан, но это была не псевдонаучная атсральная хуйня. Статья была о структуре материи и на определенном этапе рассуждений автор высказал такую мысль.
Аноним 18/11/14 Втр 14:36:53 #413 №135990 
>>135929
Да, должно и оче видно, влияет. Но уровень этого влияния загнан под шконарь вращением земляшки вокруг осяшки, обращением вокруг слонца, обращением луняшки вокруг земли и их следствиями, такими как пассаты, муссоны, ежедневные колебания довления, а также циклонами, антициклопами, тайфунами и грозами.
Аноним 18/11/14 Втр 14:39:42 #414 №135991 
>>135980
> Статья была о структуре материи
даже если ты брал оригинал статьи на архиворге, то какого ты ищешь рецензентов на дуощах.

Тезис говно/афтар - мудак
Аноним 18/11/14 Втр 15:16:13 #415 №136004 
>>135980
>>135991
Вангую, что речь шла о какой-нибудь CPT-инвариантности, но анон нихуя не понял.
Аноним 18/11/14 Втр 20:29:23 #416 №136078 
>>135857
>Суммарная тяга 18 Ньютонов
Там килоньютоны.
Аноним 19/11/14 Срд 09:50:17 #417 №136220 
>>135980
по диаграммам Фейнмана так, не?
Аноним 19/11/14 Срд 10:27:10 #418 №136222 
14163820305160.jpg
>>136220
Так.
Аноним 19/11/14 Срд 19:55:18 #419 №136396 
14164161185460.jpg
>>136078
>Там килоньютоны.

Нихуя.
Аноним 19/11/14 Срд 21:00:26 #420 №136416 
>>136396
Какое же у него ускорение?пол миллиметра в секунду всекунду? он так пять лет до луны разгонятся. А потом же еще тормозить надо.
Аноним 19/11/14 Срд 21:25:05 #421 №136430 
>>136416
Да это бред. Не может быть такая тяга при такой мощности.
Аноним 19/11/14 Срд 21:43:06 #422 №136447 
>>136430
Зависит от импульса. Хочешь получить 7000с как на пикче - снижай тягу.
Аноним 19/11/14 Срд 21:44:57 #423 №136449 
>>136430
Заебали уже ньюфаги, "не может быть, не верю", как только что-то не клеится с их убогим сознанием и мироощущением.
Аноним 19/11/14 Срд 21:45:17 #424 №136450 
>>136416
В районе нескольких недель - пары месяцев. Для груза самое оно.
Аноним 20/11/14 Чтв 02:15:34 #425 №136514 
Смотри, а если графениевые радиаторы к ТЯРДу приделать, они сильно помогут по сравнению с обычными? Или это по дефолту подразумевается и всё равно мало?
Аноним 20/11/14 Чтв 02:26:17 #426 №136515 
>>136450
Натыкался на инфу что ТЭМ будет около 5 месяцев к Луне ползти. Но это не помню для кого именно варианта, но вроде не мегаватный. Поясните: это хуета?
Аноним 20/11/14 Чтв 10:38:13 #427 №136576 
>>136515
Вполне реалистичная цифра для тяжелого аппарата. LADEE разгонялся раскачкой месяц на малой тяге, например.
Аноним 20/11/14 Чтв 16:38:24 #428 №136632 
>>129769
сказка ложь, да в ней намёк:
"...- А сколько, кстати, экипажа? – спросил он.
- Если ориентироваться на «Daedalus», то четверо, плюс два центнера груза.
- И какая у них вероятность выжить при таком раскладе? – вмешался Рон.
- Мне кажется, что высокая, - ответила она, - Ведь «мамонтовый ген» это не только удлиненная молодость, но и усиленная сопротивляемость организма.
- Просто праздник, - проворчал экс-коммандос, - Как любит говорить Уфти: «убитый вражеский боец - это не только твой вклад в победу, но и сто фунтов свежего мяса».
- Не смешно! – объявила Рибопо.
- Ну! - Рон кивнул, - Я ему тоже всегда говорю: не смешно, а он не верит. Я, вообще-то хотел спросить, что из сетевого флейма о мамонтовом гене для человека - правда, а что вранье. В серьезных популярных источниках было только про опыты с мышками.
- Так, - произнес Мак Лоу, - Начнем с того, что в середине лета ко мне на Никаупара заявились исландские неандертальцы. Колоритные ребята, кстати.

Микеле Карпини удивленно поднял брови.
- Исландские неандертальцы были у тебя в гостях?
- Да. Их интересовали долгоживущие водяные землеройки, которыми уже длительное время занимается Рохо Неи и долгоживущие тупайи, которыми занимаюсь я. Как вы понимаете, интерес касался возможности применения всего этого к человеку. Я это не рассказывал раньше, поскольку я обещал держать в секрете до некого момента.
- До момента рождения человеческих детей с этим геном? – спросила Чубби.
- Верно, - Мак Лоу кивнул, - эти исландцы упорно хотели какой-то определенности, объявленной от имени науки. Сначала они ездили к доктору Рохо Неи, и даже писали пропорции, вроде: 1 год для водяной землеройки равняется 50 лет для человека. Если землеройки с встроенным «мамонтовым геном» дожили до 8 лет и даже не выглядят постаревшими, то человек должен остаться молодым до 400 лет. Рохо выкрутился: он напоил исландцев какао, и объяснил, что между землеройкой и человеком огромная разница, и что простая пропорция тут не работает. Другое дело - тупайя, ближайший общий предок всех приматов. А тупайями занимается доктор Мак Лоу, до него всего тысяча миль на запад… Рохо не хотел их расстраивать. И я тоже. Слово за слово, и я согласился провести эксперимент с волонтерами-людьми. Это не составляло особой сложности, но я не решался проявить инициативу…
- А исландцы решились? – предположил Оскэ.
- Да. Едва я согласился, как прилетели полдюжины девчонок. Здоровые скандинавки, немного старше двадцати лет, толковые, полностью информированные…
- И теперь имеется шесть малышей с «мамонтовым геном»? – спросил Рон.
- Восемнадцать малышей, - ответил молекулярный генетик, - Они рожали тройни. В элаусестерском стиле. Сейчас это уже можно достаточно точно регулировать. Все малыши с «мамонтовым геном» родились здоровыми, мы провели целую кучу тестов, результаты нормальные. Это не доказывает, что наши малыши будут жить 400 лет, но, можно считать установленным, что мамонтовый ген не опасен для человека.
- Это уже немало, - заметил Микеле.

Мак Лоу стряхнул наросший на сигаре столбик пепла и откликнулся:.
- Немало… У нас в «Центре экстремальной биологии» на сегодняшний день лежит больше тысячи заявлений от молодых женщин, которые желают стать волонтерами следующей серии. В основном они из Канады и Скандинавии.
- И что «Центр» будет с этим делать? – спросила Чубби.
- Если у этих милых дам нет проблем со здоровьем, то «Центр» пойдет им навстречу. Понятно, что будет скандал на всю планету, но…
- Не будет большого скандала, - перебила она, - Тебя и коллег обольют помоями на нескольких TV-каналах, и все. Сейчас все заняты войной католиков с гугенотами.

- Если кого-то интересует мое чисто-дилетантское, мнение… - начал Хаген.
- Не прибедняйся, бро, - сказала Фэнг.
- …То, - продолжил он, - продав достаточному числу людей этот «мамонтовый ген», можно спокойно курить бамбук и медитировать. Все остальное, в смысле полетов к звездам и прочим галактикам, сделается само. У людей не будет другого выхода.

Мак Лоу приподнял брови и повернулся к Хагену.
- В каком смысле, не будет другого выхода?
- В психологическом, - пояснил робототехник, - Если хабитант считает, что его детям хватит кофе, сахара и булочек с кремом, это одно, а если он уверен, что на их век не хватит, то это совсем другое. В условно-западном мире гуманизм и так уже выходит из моды. Вы видели, что творится Европе, в Магрибе, и в Карибском регионе? А если еще «мамонтовый ген» у детей… Тут с хабитанта слетят последние остатки гуманизма. Он резко метнется расчищать своему потомству жизненное пространство на этой планете, потом путь к другим планетам, и так - пока палец не устанет давить на гашетку.

Экс-майор INDEMI сделала большие глаза.
- Хаген! Да ты просто экстремист!
- Что ты, тетя Чубби! Я вообще плюшевый! А вот док действительно экстремист. Без обид, док Мак, но ты сам сказал: больше тысячи мам на очереди в следующую серию. Дальше будет десять тысяч... Потом сто. Я думаю, у них в агитке уже написано, типа: Планета Земля - не зоосад для всякой консервативной фауны, а космодром для наших киндеров, которые полетят завоевывать космос от Альфы Центавра, и до самой, блин, долбанной Туманности Андромеды. Из этого - понятные скорострельные оргвыводы.
- В агитке у кого? – спросил Мак Лоу.
- У неандертальцев, - пояснила Чубби, - Ты знаешь, Мак, что подарил неандертальцам единственное, чего им не хватало для экстремизма: мощную научно - обоснованную футурологию, в которой они играют роль архитекторов будущего галактики. Одна пара мамонтовых неандертальцев… Вот, термин придумался. Одна такая пара способна на протяжении своей жизни заселить своими потомками целую планету. Увидеть двадцать поколений своих потомков: крепких, умных, смелых, сильных, позитивно-агрессивных! Настоящее нео-первобытное племя в процессе экспансии! Заселить своими потомками континент после, как бы, зачистки – это вообще плевое дело. Ты этого хотел, Мак?
- Я не особенно задумывался о социально-политическом аспекте, - ответил он, - а ты считаешь, что это очень плохо?
- Хорошо – плохо, это, в данном случае, абстракции. Для нашего региона наличие подобного племени не означает ничего принципиально нового, и уж точно ничего плохого. А вот для условно-западной цивилизации… Я не исключаю, что твоя шутка окажется последней соломинкой, которая переломит спину этому верблюду.

Доктор Мак Лоу задумался на минуту, а потом махнул рукой.
- Такая, видно, судьба у этой цивилизации. К середине каждого столетия, кто-нибудь ломает ее горбатую спину. Черт с ней..."

тов. Розов "Драйв Астарты"
Аноним 20/11/14 Чтв 19:18:06 #429 №136678 
Какой мощностью и размерами должен обладать двигатель, способный столкнуть Луну с орбиты?
Аноним 20/11/14 Чтв 20:35:46 #430 №136690 
>>136632

Ну и зачем тут эта хуйня из русфентези?
Аноним 20/11/14 Чтв 20:37:26 #431 №136691 
>>136678
Неважно. Главное чтобы скорость истечения была больше 2.38км/с, т.е. второй космической Луны, иначе никакого сталкивания не произойдет.
Аноним 20/11/14 Чтв 20:55:13 #432 №136697 
Сделал поиск по треду по слову "сигнал" - на нашло.

Как учёные отсылают команды и принимают данные с далёких космических зондов/кораблей? Вояджер, Розетта, вот это вот всё. И в каком виде посылаются/приходят эти сигналы и с какой скоростью?
Аноним 20/11/14 Чтв 20:59:36 #433 №136699 
>>136697
>в каком виде посылаются/приходят эти сигналы
В виде электромагнитных волн
>и с какой скоростью?
Со скоростью света
Аноним 20/11/14 Чтв 21:07:02 #434 №136700 
>>136699
Типа как радио?
Аноним 20/11/14 Чтв 21:08:52 #435 №136701 
>>136697
Сеть здоровых параболических антенн на земле. DSN от NASA, например. Скорость маленькая.
Аноним 20/11/14 Чтв 21:09:59 #436 №136703 
>>136701
Спасибо.
Аноним 20/11/14 Чтв 21:58:14 #437 №136720 
>>136697
Отсылаются радиосигналами со станций дальной космической связи.
>>136700
Не как, а радио.
Аноним 21/11/14 Птн 00:13:56 #438 №136725 
Реально ли построить ракету, способную вывести, скажем, десять тысяч тонн полезной нагрузки одним пуском?
Понятно, что сейчас это не надо. А вот если реально припрет - решаема ли такая задача хотя бы в теории или есть принципиальные ограничения.
Аноним 21/11/14 Птн 00:51:13 #439 №136726 
>>136725
>что сейчас это не надо.
А че, очень даже надо. Уже бы давно на Марсе жили, если бы могли такие хуевины строить.
>десять тысяч тонн
У нас проблема в том, что из химии мы выжали 99%, из того, что она может дать в теории. Нужен другой двигатель, которого нет.
Аноним 21/11/14 Птн 00:55:07 #440 №136727 
>>136697
>с какой скоростью?
Радиосистема «Вояджера» передавала поток информации со скоростью 115,2 кбит/с с Юпитера и 45 кбит/с — с Сатурна. Первоначально расчётная скорость передачи с Урана составляла лишь 4,6 кбит/с, однако её удалось повысить до 30 кбит/с, так как к тому времени ввели более чувствительные радиотелескопы на Земле, а также научились лучше сжимать данные: на определённом этапе миссии система кодирования радиосигналов была заменена на код Рида — Соломона, для чего был перепрограммирован бортовой компьютер.
Аноним 21/11/14 Птн 02:18:17 #441 №136738 
>>136725
Нельзя, ибо масса ракеты растет быстрее, чем масса нагрузки. (формула Циолковского для многоступенчатых ракет). Для 10 тыс. тонн ПН придется строить настолько титаническую йобу, что компоненты будут нагружены далеко за пределами возможностей известных материалов.
Аноним 21/11/14 Птн 09:11:13 #442 №136784 
>>136726
Ещё толком не исследованы трехкомпонентные виды топлива, графеновые баки, двигатели из сверхстойкой керамики, прямоточные двигатели итд
>>136738
Ну какое там может быть соотношение топлива к ПН? 1 к 30? 1 к 100? То есть стартовый вес подобной йобы должен быть порядка 300-1000 килотонн. Это всего лишь крупный танкер, такие вполне себе плавают по нашим морям.
Аноним 21/11/14 Птн 10:43:48 #443 №136789 
>>127536
Может ли существовать жизнь на поверхности НЗ/внутри ЧД? Как эта жизнь будет выглядеть? Могут ли НЗ/ЧД скомпенсировать свою силу притяжения центробежной(у НЗ на экваторе например) силой(вращение же оче быстрое) так чтобы человека не распидорасило на поверхности НЗ/внутри ЧД? Могут ли внутри ЧД быть стабильные орбиты?
Аноним 21/11/14 Птн 11:07:51 #444 №136793 
>>136789
>Может ли существовать жизнь на поверхности НЗ/внутри ЧД?
Нет. На поверхности НЗ не могут существовать даже молекулы, не говоря об организмах. В центре ЧД вещество, очевидно, находится в ещё более экстремальном состоянии.

>Могут ли НЗ/ЧД скомпенсировать свою силу притяжения центробежной
Нет. Если бы гравитация, держащая звезду, была компенсирована центробежной силой, значит давление вырожденного газа, излучения и прочие силы, разрывающие звезду, уже ничем не компенсированы, и звезда бы взорвалась.

>так чтобы человека не распидорасило на поверхности НЗ/внутри ЧД?
Гравитация этих объектов вообще не должна тебя волновать, ты умрёшь от чудовищного потока живительной радиации задолго до того, как приблизишься к ним, а приливные силы разорвут твоё тело задолго до того как оно упадёт на объект.

>Могут ли внутри ЧД быть стабильные орбиты?
Да. Но это не совсем то, о чём ты думаешь.
Аноним 21/11/14 Птн 11:35:58 #445 №136796 
>>136784
Ну тогда ладно, разрешаю запускать.
Аноним 21/11/14 Птн 11:39:38 #446 №136797 
>>132839
фэншуй какой-то.
Аноним 21/11/14 Птн 11:43:59 #447 №136798 
>>134757
>Даже у твоей мамки.

Почему эт мамка станет чёрной дырой? Как её не сжимай она больше массой не станет и никого притянуть не может.
Аноним 21/11/14 Птн 11:49:00 #448 №136799 
>>136396
что за генератор капель
Аноним 21/11/14 Птн 11:50:42 #449 №136801 
>>136798
Больше массой не станет, но станет меньше радиусом. Тогда гравитационная сила твоей мамки на сверхмалых дистанциях пересилит все остальные, и она коллапсирует внутрь, превратясь в ЧД. Правда очебыстро испарится, по современным представлениям.
Аноним 21/11/14 Птн 11:53:54 #450 №136803 
>>136801

Протон, нейтрон недостаточно мал и плотен, чтобы стать ЧД?
Аноним 21/11/14 Птн 11:57:12 #451 №136805 
>>136789
Роберт Форвард (известный астрофизик, изучавший НЗ и предложивший смещенные орбиты, плоскость которых не проходит через центр масс) предложил гипотетическую модель жизни на основе материала нейтронной звезды и описал это в своих Starquake и Dragon's Egg. Не знаю, переводились ли они на русский, правда. С художественной т.з. они так себе, но физмодель там высчитана очень серьезно.
Аноним 21/11/14 Птн 11:59:26 #452 №136806 
>>136803
У него исчезающе малая гравитация с его массой, на таких масштабах прочие взаимодействия берут верх, не давая ему схлопнуться.
Аноним 21/11/14 Птн 12:10:30 #453 №136812 
>>136801
Ну ок. Но нечто, сжимающее мамку, на границе мамки само уже и должно стать чёрной дырой же или даже более чёрной и более дырой. Получается не может быть маленькой дыры без большой вокруг.
Аноним 21/11/14 Птн 15:03:01 #454 №136834 
>>136812
> Но нечто, сжимающее мамку, на границе мамки само уже и должно стать чёрной дырой же или даже более чёрной и более дырой.
Все верно, поэтому мы и не наблюдаем самопроизвольного превращения мамок в черные дыры. Описана чисто гипотетическая ситуация. А ИРЛ рядовой механизм образования черных дыр - из отживших свое звезд, у которых достаточно массы чтобы стать ЧД, но уже недостаточно энергии, чтобы поддерживать себя от коллапсирования.
Аноним 21/11/14 Птн 18:13:06 #455 №136856 
>>134824
Всё там видно - только выдержку побольше взять нужно.
http://www.youtube.com/watch?v=FG0fTKAqZ5g
Аноним 21/11/14 Птн 18:28:08 #456 №136860 
>>134856
Дело в том, что в фотосфере звёзд излучение происходит тепловое переизлучение. т.е. гамма-квант, родившийся в ядре звезды и имеющий фиксированную длину волны равную разнице в массе синтезируемых элементов и субстрата, переизлучается в прозрачной для гамма-лучей мантии зоне лучистого переноса, и попадает во внешний слой, который излучает не по механизму люминесценции возбуждения-релаксации атомных оболочек, т.е. потенциальной энергии электрона, а по механизму теплового излучения - т.е. превращения колебаний и кинетической энергии всего атома в электромагнитные волны, а такой спектр, спектр чёрного тела, непрерывен.
Выше фотосферы находится хромосфера, в которой атомы избирательно поглощают на той длине волны, на которой они при возбуждении бы излучали и на непрерывный спектр фотосферы накладывается линейный спектр поглощения хромосферы.
Аноним 21/11/14 Птн 18:36:34 #457 №136863 
>>136860
Хотя я наврал - в зоне лучистого переноса квант переизлучается слоями частиц, зона для кванта непрозрачна.
https://ru.wikipedia.org/wiki/Зона_лучистого_переноса
Но суть такая - излучает в космос нагретый газ фотосферы, а греет его радиация из центра звезды, передаваемая зоной лучистого переноса.
Аноним 21/11/14 Птн 19:14:01 #458 №136873 
Будет ли в вакууме "высасываться" воздух из ануса так же как и из лёгких? А если я смажу очко скипидаром и засуну туда анальную пробку + проведу трубочки из кишки в лёгкие то сколько ещё я смогу дышать?
Аноним 21/11/14 Птн 19:18:13 #459 №136877 
>>136873
Это как раз тот случай, когда теория не может дать исчерпывающего ответа и только эмпирические
Аноним 21/11/14 Птн 19:31:49 #460 №136889 
>>136873
>проведу трубочки из кишки в лёгкие то сколько ещё я смогу дышать?
В кишках кислороду как-то маловато. Если ты не страдаешь метеоризмом, то примерно литр газа - азот, углекислый газ, метан там в основном. Или ты сначала хочешь в жопу нагнать воздушной смеси?
Аноним 21/11/14 Птн 19:58:05 #461 №136897 
Как тормозить в конечной точке?
Аноним 21/11/14 Птн 20:02:12 #462 №136900 
>>136897
В Final Destination? в конечной точке чего, ёба?
Аноним 21/11/14 Птн 20:04:46 #463 №136901 
>>136897
Левой ногой выжимаешь сцепление, правой давишь на тормоз.
Уеба, торможение - это ускорение с обратным знаком, осознав это, тебе останется повернуть корабль жопой вперед и включить двигатели
Аноним 22/11/14 Суб 10:42:56 #464 №137047 
сколько стоит слетать на Луну?
Аноним 22/11/14 Суб 11:24:11 #465 №137055 
>>137047
Если погуглить, то выплывают различные частные проекты, которые называют разные суммы, нижняя планка билета от 500 000 000 долларов.
Аноним 22/11/14 Суб 12:43:43 #466 №137070 
>>137055
Браун за 135 современных лярдов уложился
Аноним 22/11/14 Суб 13:36:37 #467 №137077 
14166525970290.png
>>137047
БЕСЦЕННО
Аноним 22/11/14 Суб 13:40:02 #468 №137078 
>>127536
Скажити посоны, а Кип Торн теперь зашкварен или всегда был шкваркой?
Аноним 22/11/14 Суб 14:03:01 #469 №137079 
>>137078
Кип Торн был и остается одним из самых выдающихся астрофизиков современности, только петухи могут в этом сомневаться.
Аноним 22/11/14 Суб 15:05:58 #470 №137086 
>>137078
> Скажити посоны, а Кип Торн теперь зашкварен или всегда был шкваркой?
> зашкварен или всегда был шкваркой?
> зашкварен … шкваркой

«Звонок» всегда событие, даже для матерого зэка. И Крест в день освобождения испытывал волнение. В общей сложности из пятидесяти пяти лет он провел за проволокой двадцать – половину сознательной жизни. Первый срок «взял» в пятнадцать – шестерик за групповой разбой с «мокряком». До совершеннолетия топтался в зоне для малолеток, где волчьи законы соблюдаются со слепым фанатизмом. Если красный цвет западло – все! Принесли предки в «дачке» помидоры ранней весной – бросай в сортир! Мать пришла на свиданку в красном платье – повернулся, сплюнул и обратно в отряд!
Правда, отца у Креста не было, а мать алкашничала и сдохла под забором, потому ему соблазнов преодолевать не приходилось, вот и порвал пасть ублюдку, жравшему тайком яблоки. На «взросляк» пришел уже с авторитетом и продолжал держаться за «закон», как за спасательный круг.
Аноним 22/11/14 Суб 15:21:43 #471 №137090 
>>137079
>>137086
но зачем он консультировал кино? Оно же всё красное!
Аноним 22/11/14 Суб 15:36:57 #472 №137093 
>>137090
Хотел и консультировал, у тебя почему жжет от этого?
Он не только консультировал, он и основу этого фильма придумал
Аноним 22/11/14 Суб 18:28:39 #473 №137131 
>>137079
Он не должен быть лучшим во всём. Вот Каспарыч тоже был лучшим шахматистом планеты, но как политик он отстой.
Аноним 22/11/14 Суб 19:39:27 #474 №137170 
Если соединить Юпитер и Сатурн, то хватит ли массы чтобы зажечь получившуюся хуитку?
Аноним 22/11/14 Суб 19:45:28 #475 №137176 
>>137170
думаю если все газовые гиганты в кучу свалить всёравно синтез в ядре не начнётся.
Аноним 22/11/14 Суб 19:49:09 #476 №137179 
>>137170
Штук 10 Юпитеров может и хватит.
Аноним 22/11/14 Суб 22:49:55 #477 №137220 
>>137170
Нет, что бы зажечь Юпитер в него надо накинуть еще 50-70 таких же Юпитеров.
Аноним 23/11/14 Вск 06:07:28 #478 №137255 
Спэйсаны, я тут сравнительно почитал про космические программы в википедии. Меркурий, восток, джемини, союз, аполлон соответственно. Так вот, количество косяков в американских программах превышает аналогичное в советских многократно. С чем это связано? С рукожопостью американцев или с сокрытием неудач советской стороной?
Аноним 23/11/14 Вск 06:15:43 #479 №137257 
>>137255
Так уж и многократно?
Единственные косяки, в которых США уверено обгоняет Советы - это завертыши среди космонавтов. В остальном никаких особенных опережений по фейлам я не вижу.
Аноним 23/11/14 Вск 06:21:43 #480 №137258 
>>137257
Ну, может показалось, может статьи писались слишком субъективно. Просто у американцев косяки в автоматике практически каждый полет, обосрамс с запуском спутника, ну и эпичное барбекю "Аполлон-1". Хотя я статью по союзам пока не осилил, может нагнали.
Аноним 23/11/14 Вск 06:34:01 #481 №137260 
>>137258
Так у наших тоже косяков было и в автоматике и вообще. Гагарин на спуске чуть не задохнулся из-за клапана, например. Леонов еле-еле в КК залез, после выхода в открытый космос.
Аполоновцев сожгли совершенно по идиотски, конечно, косяк еще на стадии проектировки, кстати, именно на стадии проектировки наши угробили Комарова, в первый раз, когда капсулу говном липким покрыли, второй обосрамс - почти моментальный выпуск запасного, и это при болтающимся основном.

Не берусь сказать, кто насрал себе больше, но неудач было много с каждой стороны.
Аноним 23/11/14 Вск 06:42:57 #482 №137262 
>>137260
>Гагарин на спуске чуть не задохнулся из-за клапана
Да, вот это кстати интересный факт. Основную мысль, которую я вынес из статей, так это то, что Юра с Нилом были настоящими мужиками в стрессовых ситуациях и лавры свои носили не зря.
Аноним 23/11/14 Вск 06:50:55 #483 №137263 
>>137262
Да они все там парни не промах. Абы кого не брали.
Аноним 23/11/14 Вск 06:54:52 #484 №137264 
>>137262
>были настоящими мужиками в стрессовых ситуациях и лавры свои носили не зря.
А то.
http://geektimes.ru/post/168937/
Аноним 23/11/14 Вск 09:00:34 #485 №137270 
>>137257
>Единственные косяки, в которых США уверено обгоняет Советы - это завертыши среди космонавтов.
Вот только в целом по космической программе Советы завернули в разы больше своих граждан, чем США. Один взрыв ракеты на площадке унёс вдвое больше, чем полвека освоения космоса американцами.
Аноним 23/11/14 Вск 09:34:21 #486 №137272 
>>137270
Тебе все печет, я смотрю?
Аноним 23/11/14 Вск 11:14:26 #487 №137281 
>>137272
У тебя детектор на отечественной элементной базе, то был мой первый пост итт.
Аноним 23/11/14 Вск 14:56:26 #488 №137308 
>>137257

Чего эт вы про завёртышей? Сегодня летят с запасного старта.
Аноним 23/11/14 Вск 20:07:29 #489 №137367 
Сосаны, а покажите снимок со спутника в максимальном на сегодняшний день разрешении и качестве.
Аноним 23/11/14 Вск 20:33:46 #490 №137376 
>>137367
Зеленый скот пост написал по этому поводу

http://zelenyikot.livejournal.com/47205.html
Аноним 24/11/14 Пнд 09:38:48 #491 №137473 
У Фэлкона как-то отказал один из движков и он всё равно вывел полезную нагрузку на орбиту.
Может ли Союз продолжить полёт при отказе одного из двигателей?
Аноним 24/11/14 Пнд 10:31:16 #492 №137484 
>>137473
Да
Аноним 24/11/14 Пнд 10:59:06 #493 №137488 
Слышал о теории, что на Землю жизнь (сложную органику) могли занести метеориты с Марса.
Но Марс же довольно массивная планета, чтобы плеваться метеоритами, это не пояс астероидов, возмущаемый гравитацией Юпитера. Как с Марса могли стартовать метеориты да ещё и попадать по Земле?
Аноним 24/11/14 Пнд 11:02:47 #494 №137490 
>>137473
Нет - пришлось бы отключать симметрично другой, а это уполовинило бы тягу.

Резервировать двигатели дорого и нецелесообразно, потому что:
1. Двигатель "выходит из строя" обычно не тихо-мирно отключаясь, а как на пик рилейтед, т.к. это охуительно нагруженный агрегат
2. Чаще всего двигатель один, лишь на первой ступени бывает больше.
3. Чтобы отключить движок и не перекосить ракету, надо симметрично отключать ещё один
4. Тяги может не хватить, а если хватит, то не дотянешь до расчетной орбиты, как и случилось с фалконом (он потерял спутник)
5. Надо брать запас в каждый полет с расчетом на фейл движка.

У Фалкона-9 такая возможность предусмотрена из-за врожденной ненадежности конструкции (аж 9 движков на первой ступени = в 9 раз выше вероятность отказа), что не соответствует стандартам NASA и они требуют от тех engine out capability и запас горючки. У Сатурна-5 была ограниченная возможность, у Н-1 вообще было 33 движка и ей было похуй на отключение двух (но не на распидорас, естественно).

Вообще, система спасения это выгодней и безопасней чем резервирование двигателей. При пилотируемых пусках юзают/юзали именно её.
Аноним 24/11/14 Пнд 11:03:23 #495 №137492 
14168162036820.jpg
>>137490
Аноним 24/11/14 Пнд 11:10:56 #496 №137493 
>>137488
Астероид пизданул по Марсу и выбил камушек. Извержение (когда ещё был вулканизм) плюнуло куском. И т.д. Даже с Земляшки не исключено что утекает материал, при её-то атмосфере.
Аноним 24/11/14 Пнд 11:17:22 #497 №137494 
>>137490
Надо же.
Я-то думал, что иметь несколько двигателей на борту, как это делает Фэлкон как раз-таки годная идея, ведь отключение одного не гарантирует отказ миссии полностью, как это и случилось.
По поводу Н-1 я думал, что несовершенная автоматика, едва ламповая, просто не могла успеть отреагировать на отказ одного из двигателей.
Система Аварийного Спасения используется только при пилотируемых полётах. ПН в виде АМС стоимостью в сотни мегабаксков ей не оснащаются по причине, как я понимаю, того, что она не расчитана на перегрузки, которые выдает САС. Поэтому резервирование двигателей выглядит более надежным методом здесь.

Ангара, как я понимаю, не сможет в такие маневры, т.к. один УРМ оснащён одним двигателем и отказ гарантирует потерю ракеты и ПН, верно?

>>137493
Довольно тяжело поверить, что падение астероида такой мощности (сопоставимо с мощнейшим ядерным взрывом или даже сильнее) не выпиливает всяческую органику как автоклав, с сотнями давлений и температур, достаточными, чтобы расплавить камень.
Аноним 24/11/14 Пнд 11:33:01 #498 №137497 
>>137494
Успевать реагировать-то она могла (лампы к этому никакого отношения не имеют), но там пидорасило движок.

Фалкон добросил одну нагрузку только за счет потери первой, если бы была одна - нечего было бы сбрасывать.

> Я-то думал, что иметь несколько двигателей на борту, как это делает Фэлкон как раз-таки годная идея, ведь отключение одного не гарантирует отказ миссии полностью, как это и случилось.
Теория надежности гласит что иметь один надежный движок в 9+ раз лучше, чем 9 надежных движков. Ну вот отключило два двигателя - остальные вынуждены усираться на неоптимальных режимах. Это надо брать запас горючки же, предусматривать нештатную нагрузку на здоровые движки, переусложнять систему, сплошной риск везде, а если все пройдет штатно то лишняя нагрузка. В дополнение к тому что система и так уже переусложнена, и отключение двигателей до распидораса вовсе не гарантировано.

Короче, 1 движок всегда будет надежней чем 9, даже с engine out capability. EOC это необходимый костыль.

> Ангара, как я понимаю, не сможет в такие маневры, т.к. один УРМ оснащён одним двигателем и отказ гарантирует потерю ракеты и ПН, верно?
А-7 теоретически может и смогла бы. В плане согласования СУ может и можно что-то придумать было бы, хз.
Аноним 24/11/14 Пнд 11:38:37 #499 №137498 
>>137494
> Я-то думал, что иметь несколько двигателей на борту, как это делает Фэлкон
Он имеет их из других соображений - унификация, дешевизна, повышенный ресурс, простота управления тягой на посадке своим ходом, и т.п. Расплачиваясь снижением надежности.
Аноним 24/11/14 Пнд 13:38:59 #500 №137519 
>>137497
>Теория надежности гласит что иметь один надежный движок в 9+ раз лучше, чем 9 надежных движков
Какая-то хуета эта твоя теория надежности. Зачем тогда у бронетехники дохуя колёс/катков? Оставить 4 или вообще 1, надёжно же. Зачем жёсткие диски объединяют в RAID?
Аноним 24/11/14 Пнд 13:46:52 #501 №137522 
>>137519
Ты какой-то имбецил, нахуй ты бронетехнику приплел с рэйдами?
Аноним 24/11/14 Пнд 13:59:28 #502 №137525 
>>137522
Во всех отраслях концепция "много говнодеталей" надёжнее, чем "одна опиздохуительно дорогая суперпупер надёжная деталь", а блять в космонавтике-то почему не так?
Аноним 24/11/14 Пнд 14:16:31 #503 №137529 
>>137525
>Во всех отраслях концепция "много говнодеталей" надёжнее, чем "одна опиздохуительно дорогая суперпупер надёжная деталь
Нет.
>а блять в космонавтике-то почему не так?

Потому нигде не так и ракетостроение - это не бронетанковые клинья с рэйдами на борту, на, читай Бориса Ивановича, до полного просветления, он все по хардкору расписал, за надежность, дублирование и отказы, мнимые и реальные.
http://www.buran.ru/htm/27-3.htm
Аноним 24/11/14 Пнд 15:00:31 #504 №137534 
>>137525
Теория вероятностей и зависимые/независимые события, только и всего.
Аноним 24/11/14 Пнд 15:10:13 #505 №137535 
>>137519
Затем что винты в RAID при отказе не взрываются, разнося вместе с собой остальные и пол-датацентра впридачу.
Аноним 24/11/14 Пнд 15:24:18 #506 №137536 
>>137535
Про взрыв принимается, но это не единственная возможна неисправность двигателя. Бывают заклинившие насосы, пиздящие датчики и т.д. Посмотри новости, всё что ебануло за последнее время - это крушения и самоподрывы, такого чтобы непосредственно взрыв двигателя распидорасил ракету - это редкость.
Аноним 24/11/14 Пнд 15:34:50 #507 №137538 
>>137536
Мы говорим именно об отказе двигателя, а не СУ или еще чего-либо. А они в основном кончаются взрывом. Диагностировать проблему на лету, а тем более остановить двигатель возможно очень редко, например в окислительном тракте любой дефект ТНА приводит к катастрофическому развитию событий в пределах сотен микросекунд.

И это только одно соображение, связка движков всегда сложней одного большого, по разным причинам.
Аноним 24/11/14 Пнд 16:08:09 #508 №137544 
>>137494
> Довольно тяжело поверить, что падение астероида такой мощности (сопоставимо с мощнейшим ядерным взрывом или даже сильнее) не выпиливает всяческую органику как автоклав, с сотнями давлений и температур, достаточными, чтобы расплавить камень.
Да банальный маленький метеорит с кирпич размером, зачем йоба-астероид. Вон в Бионе-М кажется проверили, что микроорганизмы могут выжить в пористой поверхности снаружи капсулы при реентри, а уж тем более просто органика и тем более в маленьком булыжнике.
Аноним 25/11/14 Втр 00:01:13 #509 №137680 
Спейсач, помоги. Недавно обмазался рандомным фильмом, найденным в интернете "Путешествие к краю вселенной", он оказал на меня очень сильное впечатление, всякие интерстеллары и в сравнение не идут. Так вот, подскажите подобного научно-популярного контента
Аноним 25/11/14 Втр 07:10:59 #510 №137709 
>>137519
>Зачем тогда у бронетехники дохуя колёс/катков? Оставить 4 или вообще 1, надёжно же.
Затем, что это военная техника, ебанавт, условия её эксплуатации подразумевают враждебную среду и гарантированные повреждения узлов.
>Зачем жёсткие диски объединяют в RAID?
Затем, что могут себе позволить, а информация зачастую стоит на порядки дороже носителя. Ты свою порнуху на RAID 1 массиве хранишь?
Ты хоть с теорией ознакомься прежде чем кудахтать, уманетарий.
Аноним 25/11/14 Втр 08:18:04 #511 №137711 
антон, а есть ли на луне вода, и можно ли её тераформить?
Аноним 25/11/14 Втр 08:48:09 #512 №137712 
>>137711
Вода на луне содержится в грунте.
Терраформировать воду нель-
А, луну?
https://ru.wikipedia.org/wiki/Колонизация_Луны
Скорее перевалочный пункт запилят, купола, базы, все дела. Со своей атмосферой и магнитным полем гемор.
Аноним 25/11/14 Втр 18:00:23 #513 №137767 
>>137712
Все статьи про терраформирование на рукипедии, как им и положено, писаны какими-то фриками с форумчиков и двачей. Кольцо построим - магнитосфера возбудится, вообще охуеть.
Аноним 28/11/14 Птн 02:12:35 #514 №138344 
с какой скоростью распространяется гравитация?
Со скоростью света или с более высокой и вплоть до бесконечной?

Разница примерно вот в чём. Если со скоростью света, то, допустим, если Солнце внезапно исчезнет, улетит, будет чем-то сбито и т.д., то до Земли гравитация дойдёт мгновенно или через 500 секунд? Если мгновенно, то Земля дальше полетит по прямой по инерции, а если гравитация дойдёт со скоростью света, то земля ещё 500 секунд будет огибать круг вокруг точки, где было Солнце, а только потом полетит свободно по инерции дальше.

Поняли в чём разница и в чём гипотетический эксперимент как это проверить? Так вот.

Если вы считаете что с бесконечной скоростью, то почему информация не может распространяться быстрее скорости света? Почему нельзя при помощи гравитации передать инфо быстрее? Например, если Земля полетит сразу по прямой или будет лететь 500 секунд по орбите, то траекторию можно замерить не через 500 секунд а сразу. А это уже один из способов передачи информации.

Если же наоборот, гравитация ограничена скоростью света, то почему чёрные дыры притягивают и их можно определить по гравитации? Ведь их не может покинуть свет и электромагнитное взаимодействие. Как и любое поле, распространяющееся со скоростью не выше световой. Получается, гравитация тоже не может их покинуть, а тогда чёрная дыра, даже если бы и существовала, была бы не обнаружима, не имела бы массы, потому что не могла бы распространить гравитацию, затягивая гравитацию в саму себя как и свет. То есть для внешнего наблюдателя чёрная дыра должна скорее была ба быть похожей на пустую область пространства, на войд. Но она имеет массу.

Где я не прав? Как разрешается это противоречие. Простите за дебильный вопрос. Спасибо за ответ.
Аноним 28/11/14 Птн 22:25:43 #515 №138590 
>>138344
> если Солнце внезапно исчезнет
Оно так не сделает же. Вот в чем.
Аноним 23/03/15 Пнд 22:55:54 #516 №162626 
>>137537
Перекат.
comments powered by Disqus

Отзывы и предложения